quiz hematology oncology part 1 of 2

84
Hematology & Oncology – Part 1 14Mar2009 DO NOT DISTRIBUTE - 1 - Hematology/Oncology #1 – Embryology 1) Which of the following does NOT function in embryonic hemopoiesis (hemangioblasts as blood cell precursors)? a) Yolk sac b) Liver c) Heart d) Bone Marrow e) Spleen 2) The first blood islands appear in the mesoderm surrounding the wall of the yolk sac at approximately how many weeks of embryonic development? a) 3-weeks b) 9-weeks c) 15-weeks d) 27-weeks e) 37-weeks 3) What structures could take over hemopoietic function in a child when the usual hematopoietic organs are incapable? a) Heart and lungs b) Kidneys and spleen c) Thymus and kidneys d) Brain and heart e) Spleen and liver f) None (lethal) Hematology/Oncology #2 – Histology 1) Which of the following is NOT true of the function of blood and plasma? a) Supply of oxygen and nutrients to tissues b) Coagulation and immunologic functions c) Hydraulic functions with blood volume of about 6.0L d) Regulation of body pH to about 7.0, neutral e) Messenger functions (hormones) and removal of waste 2) Which of the following is NOT true of the structure and function of erythrocytes (red blood cells) and hemoglobin? a) Hemoglobin is comprised of 4 polypeptide chains with a heme b) Red blood cells are biconcave in shape, maintained by membrane proteins c) Peripheral membrane proteins are on the outer surface of the cell membrane d) Integral membrane proteins make up the lipid bilayer e) Red blood cells (RBCs) have a life span of 120 days with most being phagocytized by macrophages in the liver, spleen, and bone marrow 3.1) Which of the following is NOT true for a patient with type B- blood (group B and negative Rh factor)? a) They have a type B surface antigen b) They have anti-A type antibodies c) They can donate to anyone with type B or AB blood regardless of Rh factor d) They could receive blood from a patient with type O- blood e) They could receive blood from a patient with type O+ blood

Upload: medshare

Post on 12-Nov-2014

5.105 views

Category:

Documents


6 download

DESCRIPTION

Quiz Hematology Oncology Part 1 of 2

TRANSCRIPT

Page 1: Quiz Hematology Oncology Part 1 of 2

Hematology & Oncology – Part 1 14Mar2009

DO NOT DISTRIBUTE - 1 -

Hematology/Oncology #1 – Embryology

1) Which of the following does NOT function in embryonic hemopoiesis

(hemangioblasts as blood cell precursors)?

a) Yolk sac

b) Liver

c) Heart

d) Bone Marrow

e) Spleen

2) The first blood islands appear in the mesoderm surrounding the wall of the yolk sac at

approximately how many weeks of embryonic development?

a) 3-weeks

b) 9-weeks

c) 15-weeks

d) 27-weeks

e) 37-weeks

3) What structures could take over hemopoietic function in a child when the usual

hematopoietic organs are incapable?

a) Heart and lungs

b) Kidneys and spleen

c) Thymus and kidneys

d) Brain and heart

e) Spleen and liver

f) None (lethal)

Hematology/Oncology #2 – Histology

1) Which of the following is NOT true of the function of blood and plasma?

a) Supply of oxygen and nutrients to tissues

b) Coagulation and immunologic functions

c) Hydraulic functions with blood volume of about 6.0L

d) Regulation of body pH to about 7.0, neutral

e) Messenger functions (hormones) and removal of waste

2) Which of the following is NOT true of the structure and function of erythrocytes (red

blood cells) and hemoglobin?

a) Hemoglobin is comprised of 4 polypeptide chains with a heme

b) Red blood cells are biconcave in shape, maintained by membrane proteins

c) Peripheral membrane proteins are on the outer surface of the cell membrane

d) Integral membrane proteins make up the lipid bilayer

e) Red blood cells (RBCs) have a life span of 120 days with most being

phagocytized by macrophages in the liver, spleen, and bone marrow

3.1) Which of the following is NOT true for a patient with type B- blood (group B and

negative Rh factor)?

a) They have a type B surface antigen

b) They have anti-A type antibodies

c) They can donate to anyone with type B or AB blood regardless of Rh factor

d) They could receive blood from a patient with type O- blood

e) They could receive blood from a patient with type O+ blood

Page 2: Quiz Hematology Oncology Part 1 of 2

Hematology & Oncology – Part 1 14Mar2009

DO NOT DISTRIBUTE - 2 -

3.2) What surface antigen is present in people that have type O blood?

a) A antigen

b) B antigen

c) A and B antigen

d) A or B antigen

e) No antigen

4.1) Which of the following leukocytes is the largest and partially degrades antigens to

present the fragments on MHC II for helper CD4 T-cell recognition?

a) Neutrophils

b) Lymphocytes

c) Monocytes

d) Eosinophils

e) Basophils

4.2) Which of the following leukocytes is azurophilic (lysosomes) and is associated with

allergic reactions, parasitic infections, and chronic inflammation?

a) Neutrophils

b) Lymphocytes

c) Monocytes

d) Eosinophils

e) Basophils

4.3) Which of the following leukocytes is multi-lobular, azurophilic (bactericidal

myeloperoxidase), is the first to enter damaged areas, activates phagocytes at the site of

inflammation, and secrets pyrogenic IL-1?

a) Neutrophils

b) Lymphocytes

c) Monocytes

d) Eosinophils

e) Basophils

4.4) Which of the following leukocytes is azurophilic (lysosomes) and contains heparin,

histamine, heparan sulfate, and leukotrienes?

a) Neutrophils

b) Lymphocytes

c) Monocytes

d) Eosinophils

e) Basophils

4.5) Which of the following leukocytes is the main cell of the immune system and

differentiates into T-cells (helper CD4, cytotoxic CD8), antibody producing B-cells, and

natural killer (NK) cells, which attack certain viruses and tumor cells?

a) Neutrophils

b) Lymphocytes

c) Monocytes

d) Eosinophils

e) Basophils

5.1) Which of the following is identified by the presence of CD34+ surface marker

protein?

a) Pleuripotent stem cell (PPSS)

Page 3: Quiz Hematology Oncology Part 1 of 2

Hematology & Oncology – Part 1 14Mar2009

DO NOT DISTRIBUTE - 3 -

b) Colony forming cells (progenitor, CFU)

c) Multipotent stem cell

d) Basophilic erythroblast

e) Polychromatophilic erythroblast

f) Orthochromatic erythroblast (normoblast)

g) Polychromatophilic erythrocyte

5.2) Which of the following is under the influence of erythropoietin, IL-3, and IL-4?

a) Pleuripotent stem cell (PPSS)

b) Colony forming cells (progenitor, CFU)

c) Multipotent stem cell

d) Basophilic erythroblast

e) Polychromatophilic erythroblast

f) Orthochromatic erythroblast (normoblast)

g) Polychromatophilic erythrocyte

5.3) Which of the following produces acidophilia (pink) due to hemoglobin staining and

basophilia (purple) due to ribosome staining?

a) Pleuripotent stem cell (PPSS)

b) Colony forming cells (progenitor, CFU)

c) Multipotent stem cell

d) Basophilic erythroblast

e) Polychromatophilic erythroblast

f) Orthochromatic erythroblast (normoblast)

g) Polychromatophilic erythrocyte

6) Which of the following has a densely stained nucleus with eosiniphilic (acidophilic)

cytoplasm due to large amounts of hemoglobin, is no longer capable of division, and

extrudes its nucleus from the cell?

a) Pleuripotent stem cell (PPSS)

b) Colony forming cells (progenitor, CFU)

c) Multipotent stem cell

d) Basophilic erythroblast

e) Polychromatophilic erythroblast

f) Orthochromatic erythroblast (normoblast)

g) Polychromatophilic erythrocyte

7.1) Which of the following is described as becoming more heterochromic, deeply

indented, and forming a kidney bean shaped structure with neutrophils granules to

azurophilic granules in a 2-to-1 ratio?

a) Myeloblast

b) Promyelocyte

c) Myelocytes

d) Metamyelocytes

e) Band/segmented stage

7.2) Which of the following has a euchromic spherical nucleus with agranular cytoplasm

that stains intensely basophilic and is the earliest recognizable neutrophils precursor cell

in bone marrow?

a) Myeloblast

b) Promyelocyte

Page 4: Quiz Hematology Oncology Part 1 of 2

Hematology & Oncology – Part 1 14Mar2009

DO NOT DISTRIBUTE - 4 -

c) Myelocytes

d) Metamyelocytes

e) Band/segmented stage (polymorphonuclear neutrophil)

7.3) Which of the following describes when the nucleus elongates with nearly uniform

width giving it a horseshoe like appearance?

a) Myeloblast

b) Promyelocyte

c) Myelocytes

d) Metamyelocytes

e) Band/segmented stage (polymorphonuclear neutrophil)

8.1) Which of the following cytokines (signaling proteins) is seen specifically in the

development of monocytes and not in the differentiation of basophils or neutrophils?

a) SCF (Stem cell factor)

b) M-CSF (Macrophage colony-stimulating factor)

c) IL-6 (Interleukin 6)

d) IL-3 (Interleukin 3)

e) GM-CSF (Granulocyte macrophage colony-stimulating factor)

8.2) A complete blood count (CBC) that returned with high CD14+ and CD16+,

monocytes, is most indicative of which of the following?

a) Parasitic infection

b) Glucocorticoid therapy

c) Sepsis

d) Hemolytic trauma

e) Anemia

9) Which of the following signaling cytokines is NOT involved in CFU-GEMM (colony-

forming unit granulocyte erythrocyte monocyte macrophage) stimulation to develop into

megakaryoblasts (CF-Meg), which produce platelets (thrombocytes)?

a) SCF (Stem cell factor)

b) Tpo (Thrombopoietin)

c) IL-3 (Interleukin 3)

d) G-CSF (Granulocyte colony-stimulating factor)

e) GM-CSF (Granulocyte macrophage colony-stimulating factor)

10) Lymphocytes begin as precursor cells in the bone marrow called transitional cells.

Multipotential lymphoid stem cells (CFU-L) that are designated to be cells leave the bone

marrow and travel to which area to complete differentiation?

a) Brain

b) Spleen

c) Kidney

d) Thymus

e) Thyroid

11) Which of the following describes yellow bone marrow?

a) Occupies the space in red marrow that is normally taken up by capillary

b) Secretes colony-stimulating factors (CSFs) to play a role in cell differentiation

c) Is no longer hematopoietic and can be found in long bones and phalanges

d) Is not able to revert to red marrow even in the case of major blood loss

e) Is active bone marrows as it contains blood cells and magakaryocytes

Page 5: Quiz Hematology Oncology Part 1 of 2

Hematology & Oncology – Part 1 14Mar2009

DO NOT DISTRIBUTE - 5 -

12.1) What type of cells are shown here?

a) Granulocytes

b) Agranulocytes

c) Thrombocytes

d) Erythrocytes

e) None of the above

12.2) What type of cell is shown here?

a) Neutrophil

b) Lymphocyte

c) Monocyte

d) Macrophage

e) Basophil

12.3) What type of cell is shown here?

a) Neutrophil

b) Lymphocyte

c) Monocyte

d) Eosinophil

e) Basophil

12.4) What type of cell is shown here?

a) Neutrophil

b) Lymphocyte

c) Monocyte

d) Eosinophil

e) Basophil

12.5) What type of cell is shown here?

a) Neutrophil

b) Lymphocyte

c) Monocyte

d) Eosinophil

e) Basophil

12.6) What type of cell is shown here?

a) Neutrophil

b) Lymphocyte

c) Monocyte

d) Macrophage

e) Erythrocyte

f) None of the above

12.7) What type of cell is shown here?

a) Neutrophil

b) Lymphocyte

c) Monocyte

d) Macrophage

e) Erythrocyte

f) Thrombocyte

g) None of the above

Page 6: Quiz Hematology Oncology Part 1 of 2

Hematology & Oncology – Part 1 14Mar2009

DO NOT DISTRIBUTE - 6 -

Hematology/Oncology #3 – Biochemistry: Overview

1) Sickle cell hemoglobin (HbS) has 2 normal alpha chains and 2 beta chains with sickle

cell variant. What point mutation occurs in these individuals at position 6 of the beta

chain?

a) Gly to Val

b) Val to Phe

c) Glu to Gly

d) Ser to Asn

e) Glu to Val

2) Which of the following is NOT true in the synthesis of heme?

a) Glycine and succinyl CoA condense to form delta-aminolevulinic acid (ALA)

b) Formation of delta-ALA requires delta-ALA reductase

c) Vitamin B6, pyridoxal phosphate, is an important cofactor

d) Protoporphyrin IX is formed from 4 porphobilinogens and binds iron

e) Heme stimulates the synthesis of the protein globin

3) What is the most common form of hemoglobin, seen in 95% of adults?

a) Alpha2, Beta2

b) Alpha2, Delta2

c) Alpha2, Gamma2

d) Alpha2, Epsilon2

e) Zeta2, Epsilon2

4) What is heme degraded to?

a) Albumin

b) Bilirubin

c) Cholesterol

d) Alph1-antitrypsin

e) Immunoglobulins

5) A deficiency in folate (Vitamin B9) or cobalamin (Vitamin B12) will result in

increased homocysteine, which results in:

a) Sepsis

b) Shock

c) Anemia

d) Increased bleeding

e) High blood pressure

6) Translation is the synthesis of:

a) RNA on a DNA template

b) Protein using mRNA as a guide

c) DNA on a DNA template

d) Ribosomal protein using tRNA

7) Factors VII, IX, X, and prothrombin contain a domain in which one or more glutamate

residues are carboxylated to form gamma-carboxyglutamate in a reaction requiring:

a) Vitamin A

b) Vitamin B9

c) Vitamin B12

d) Vitamin E

e) Vitamin K

Page 7: Quiz Hematology Oncology Part 1 of 2

Hematology & Oncology – Part 1 14Mar2009

DO NOT DISTRIBUTE - 7 -

Hematology/Oncology #4 – Biochemistry: Red Blood Cells

1) Which of the following does NOT arise from a multipotent myeloid stem cell (CFU-

GEMM)?

a) Megakaryocyte

b) Macrophage

c) Plasma cell

d) Neutrophil

e) Eosinophil

2) Which of the following is NOT true of erythrocytes?

a) Red blood cells (RBCs) are the most numerous cells in the blood

b) Mature RBCs contain no organelles, particularly no mitochondria

c) RBCs can synthesize protein necessary for their survival

d) RBC cytoplasm has heme and enzymes to prevent and repair oxidative damage

e) RBCs produce 2,3-BPG, which helps with O2 unloading at tissues

3) The pentose phosphate pathway (PPP) in RBCs metabolizes about 5-10% of the

glucose entering the cell and produces NADPH. Glucose-6-phosphate dehydrogenase

(G6PD) deficiency stops this pathway and thus would cause which of the following,

giving schistocytes (helmet-shaped or triangular shaped red blood cells)?

a) Thrombophilia

b) Hemolytic anemia

c) Megaloblastic anemia

d) Sickle cell anemia

e) Leukocytopenia

4) How do red blood cells generate energy?

a) Oxidation of carbohydrate, fat, protein, and alcohol

b) Aerobic metabolism of glucose into pyruvate

c) Breakdown of glucose via glucose decarboxylase

d) Anaerobic metabolism of glucose to lactic acid

e) Oxidative phosphorylation

5) Deficiency in which of the following enzymes in the heme pathway would lead to an

erythropoeitic porphyria, manifesting with skin lesions and light sensitivity?

a) Delta-aminolevulinate (ALA) synthase

b) Delta-aminolevulinate (ALA) dehydratase

c) Hydroxymethylbilan (HMB) synthase (or PBG deaminase)

d) Uroporphyrinogen (URO) decarboxylase

e) Coproporphyrinogen (COPRO) oxidase

6) A patient presents with difficulty breathing and general cyanosis. Previous genetic

testing revealed a condition where His F8 was replaced with tyrosine, with a deficiency

in the enzyme diaphorase I. Which of the following is most likely?

a) Heterozygous HbS – Sickle cell trait

b) Homozygous HbS – Sickle cell anemia

c) Heterozygous HbM – Methemoglobinemia

d) Homozygous HbC – Crystallization trait

e) Heterozygous HbC – Crystallization anemia

Page 8: Quiz Hematology Oncology Part 1 of 2

Hematology & Oncology – Part 1 14Mar2009

DO NOT DISTRIBUTE - 8 -

7) A comparison of which of the following would give a measure of intracellular

oxidative stress?

a) Ascorbic acid (vitamin C) and vitamin E

b) NADPH, superoxide (O2-), and superoxide dismutase

c) Glutathione (GSH) and oxidized glutathione (GSSH)

d) Hydrogen peroxide and peroxyl radicals

e) Hydroxyl radicals and myeloperoxidase (neutrophils)

f) Catalase and hydrogen peroxide

8.1) G6PD creates which of the following?

a) Succinyl-CoA

b) GDP

c) Pyruvate

d) NADPH

e) NAD+

8.2) Which of the following is the major red blood cell membrane (cytoskeleton) protein,

giving the membrane much of its flexibility?

a) Actin

b) Ankyrin

c) Spectrin

d) Protein 4.1

e) Band 3 protein

8.3) Splenomegaly is found during a routine physical exam. Blood tests are ordered and

return showing puffy, enlarged red blood cells with hemolytic anemia. An osmotic

fragility test is positive. The patient is consulted on a splenectomy. Which of the

following is most likely?

a) Iron deficiency anemia

b) Megaloblastic anemia

c) Aplastic anemia

d) Hereditary sideroblastic anemia

e) Hereditary spherocytosis

9) Which of the following blood types would be considered a universal donor and would

thus agglutinate every blood type but their own due to the presence of blood serum IgM

antibodies?

a) O+

b) O-

c) AB+

d) AB-

e) None of the above

10) Which of the following blood types would be considered a universal recipient as their

blood serum does not contain any antibodies against blood types?

a) O+

b) O-

c) AB+

d) AB-

e) None of the above

Page 9: Quiz Hematology Oncology Part 1 of 2

Hematology & Oncology – Part 1 14Mar2009

DO NOT DISTRIBUTE - 9 -

11.1) Which of the following forms of anemia is due to vitamin B deficiency and not

directly a heme and iron problem?

a) Megaloblastic anemia

b) Iron deficiency anemia

c) Anemia of chronic disease

d) Sideroblastic anemia

e) Thalassemias

11.2) Which of the following types of anemia is generally normocytic?

a) Pernicious anemia

b) Sideroblastic anemia

c) Anemia of chronic disease

d) Iron deficiency anemia

e) Sickle cell anemia

11.3) What type of anemia is associated with a G6PD deficiency?

a) Pernicious

b) Hemolytic

c) Megaloblastic

d) Iron deficiency

e) Sideroblastic

12.1) What type of anemia leads to the inability to maintain the activity of the Na+/K+

ATPase pump resulting in increased intracellular Na+ and cell lysis?

a) Deficiency of iron

b) Deficiency of folic acid

c) Deficiency of vitamin B12

d) Deficiency of pyruvate kinase

e) Deficiency of G6PD

12.2) Damage to the duodenum, preventing nutrient absorption, would cause which of the

following forms of anemia?

a) Deficiency of iron

b) Deficiency of folic acid

c) Deficiency of vitamin B12

d) Deficiency of pyruvate kinase

e) Deficiency of G6PD

12.3) Damage to the ileum, preventing nutrient absorption, would cause which of the

following forms of anemia?

a) Deficiency of iron

b) Deficiency of folic acid

c) Deficiency of vitamin B12

d) Deficiency of pyruvate kinase

e) Deficiency of G6PD

12.4) Damage to the jejunum, preventing nutrient absorption, would cause which of the

following forms of anemia?

a) Deficiency of iron

b) Deficiency of folic acid

c) Deficiency of vitamin B12

d) Deficiency of pyruvate kinase

Page 10: Quiz Hematology Oncology Part 1 of 2

Hematology & Oncology – Part 1 14Mar2009

DO NOT DISTRIBUTE - 10 -

e) Deficiency of G6PD

12.5) Mutations in the PIG-A gene, affecting the synthesis of GPI-anchored proteins, lead

to what form of anemia?

a) Methemoglobinemia

b) Sickle cell anemia

c) Alpha-thalassemia

d) Beta-thalassemia

e) Paroxysmal nocturnal hemoglobinemia

13) Alpha-thalassemias are most commonly caused by ____ and beta-thalassemias are

most commonly caused by ____.

a) Insertions; Deletions

b) Deletions; Point mutations

c) Point mutations; Repeat sequence mutation

d) Repeat sequence mutation; Insertions

e) Point mutations; Deletions

Hematology/Oncology #5 – Biochemistry: White Blood Cells

1) Neutrophils and other phagocytic cells engulf bacteria and rapidly increase oxygen

consumption producing large amounts of free radicals and leading to respiratory burst.

The superoxide ion plays a role in killing bacteria and is formed via:

a) G6P reductase

b) G6P dehydrogenase

c) G6P synthase

d) NADPH oxidase

e) NADPH reductase

2) What enzymes is defective in patients with chronic granulomatosis disease, which

manifests as a multisystem disorder characterized by recurrent infections?

a) Phagocyte NADPH oxidase (PHOX)

b) Angiotensin converting enzyme (ACE)

c) Glucose-6-phosphate dehydrogenase (G6PD)

d) Cytochrome b5 reductase

e) Pyruvate kinase

Hematology/Oncology #6 – Biochemistry: Blood Coagulation

1.1) What coagulation factor marks the beginning of the final common pathway where

the intrinsic (damaged surface) and extrinsic (trauma) pathways merge?

a) Factor I (fibrinogen)

b) Factor II (prothrombin)

c) Factor X (Stuart-Prower)

d) Factor IX (Christmas)

e) Factor VII (stable)

1.2) What two coagulation factors are considered cofactors that bind to other factors?

a) I & II

b) VII & XIII

c) XII & XI

d) V & VIII

Page 11: Quiz Hematology Oncology Part 1 of 2

Hematology & Oncology – Part 1 14Mar2009

DO NOT DISTRIBUTE - 11 -

e) IX & X

1.3) Which of the following is NOT a serine protease factor activated in the cascade?

a) Fibrinogen

b) Prothrombin

c) Factor VII (stable)

d) Factor IX (Christmas)

e) Factor X (Stuart-Prower)

f) Factor XI (plasma thromboplastin antecedant)

g) Factor XII (Hageman factor)

1.4) Which of the following is considered the key process in blood coagulation?

a) Activation of prothrombin to thrombin

b) Activation of factor VII to VIIa

c) Activation of factor XII to XIIa

d) Activation of factor XI to XIa

e) Activation of factor X to Xa

1.5) Which of the following is the most important naturally occurring anti-thrombin in

plasma as it inhibits many of the factors of the clotting cascade?

a) Alpha1-antitrypsin

b) Alpha2-macroglobulin

c) Antithrombin III

d) Heparin cofactor II

e) None of the above

2) Which of the following is NOT a stage of clotting hemostasis (arrest of bleeding)?

a) Clot formation

b) Vascular dilation

c) Clot dissolution

d) Formation of platelet plug

3.1) Formation of a platelet plug involves adhesion, activation, and aggregation. Which

of the following molecules that are released on platelet activation is responsible for

mediating the binding of platelets to subendothelial tissue, stabilizing factor VIII?

a) Von Willebrand factor

b) ADP, thromboxane A2

c) Coagulation factors I & V

d) 5-OH tryptamine (serotonin)

e) Platelet-derived growth factor (PDGF)

3.2) Which of the following molecules released on platelet activation is responsible for

promoting vasoconstriction?

a) Von Willebrand factor

b) ADP, thromboxane A2

c) Coagulation factors I & V

d) 5-OH tryptamine (serotonin)

e) Platelet-derived growth factor (PDGF)

4.1) What is the mechanism of anticoagulation for aspirin?

a) Potentiation of antithrombin III

b) Potentiation of heparin cofactor II

c) Irreversible inhibition of thromboxane A2 formation

Page 12: Quiz Hematology Oncology Part 1 of 2

Hematology & Oncology – Part 1 14Mar2009

DO NOT DISTRIBUTE - 12 -

d) Irreversible inhibition of proteins C and S with no vitamin K action

e) Inhibition of vitamin K-dependent reactions of I, II, VII, IX, and proteins C, S

4.2) What is the mechanism of anticoagulation for heparin?

a) Potentiation of antithrombin III

b) Potentiation of heparin cofactor II

c) Irreversible inhibition of thromboxane A2 formation

d) Irreversible inhibition of proteins C and S with no vitamin K action

e) Inhibition of vitamin K-dependent reactions of I, II, VII, IX, and proteins C, S

4.3) What is the mechanism of anticoagulation for warfarin (Coumadin, Dicumarol)?

a) Potentiation of antithrombin III

b) Potentiation of heparin cofactor II

c) Irreversible inhibition of thromboxane A2 formation

d) Irreversible inhibition of proteins C and S with no vitamin K action

e) Inhibition of vitamin K-dependent reactions of I, II, VII, IX, and proteins C, S

5) The fibrinolytic drugs (“clot busters”) tissue plasminogen activator (tPA),

streptokinase, and urokinase affect plasminogen, with streptokinase being the least

selective. Which of the following describes their mechanism?

a) Conversion of fibrinogen to fibrin

b) Release of stored plasminogen from the liver

c) Causes the secretion of plasmin from cells

d) Activation of the zymogen plasminogen to plasmin

e) Deactivation of the enzyme responsible for degrading fibrin to fibrinogen

6) Which of the following is NOT a method of blood clot inhibition that exists on the

surface of normal, intact endothelial cells?

a) Synthesis of prostacyclin (PGI2)

b) Use of ADPase to hydrolyze ADP

c) Synthesis of heparan sulfate

d) Synthesis of von Willebrand factor

e) Use of plasminogen activators

7.1) A 7-year-old African American male is brought to the Emergency Room because of

weakness and the spontaneous appearance of painful swelling of both knee joints. An X-

linked recessive disorder, hemophilia A, is diagnosed. What clotting factor is deficient?

a) Factor XII

b) Factor XI

c) Factor IX

d) Factor X

e) Factor VIII

7.2) What factor is deficient in hemophilia B, which presents clinically almost identically

to hemophilia A?

a) Factor XII

b) Factor XI

c) Factor IX

d) Factor X

e) Factor VIII

7.3) A 12-year-old white female is brought to the Emergency Room because of

uncontrollable bleeding following a tooth extraction. She has a history of prolonged

Page 13: Quiz Hematology Oncology Part 1 of 2

Hematology & Oncology – Part 1 14Mar2009

DO NOT DISTRIBUTE - 13 -

bleeding and her father has a bleeding disorder. It is found that she has a lack of von

Willebrand factor. This results in the rapid clearing of what clotting factor?

a) Factor XII

b) Factor IX

c) Factor VIII

d) Factor II

e) Factor I

7.4) Heparin is able to change the shape of platelets, thus tagging them for destruction

and leading to a reduction in the number of circulating platelets. What would this be

called?

a) Anemia

b) Leukopenia

c) Pancytopenia

d) Thrombophilia

e) Thrombocytopenia

7.5) A newborn presents with apparent hemorrhagic disease. Physical exam reveals birth

defects including underdevelopment of the face, and spontaneous bleeding under the

skin. Prothrombin time is measured at several minutes (expected is 13 seconds). The

parents have no history of bleeding problems. Which of the following is most likely?

a) Hemophilia A

b) Hemophilia B

c) Von Willebrand disease

d) Thrombocytopenia

e) Vitamin K deficiency

Hematology/Oncology #7 – Biochemistry: Plasma Proteins

1) Which of the following is NOT a major function of blood?

a) Coagulation

b) Transport of food material

c) Synthesis of hormones

d) Regulation of water balance

e) Respiration and defense

2) Albumin is considered what type of plasma protein?

a) Antiprotease

b) Hormone

c) Enzyme

d) Blood clotting

e) Transport or binding

3) IL-1 is the principle stimulator of the synthesis of acute phase proteins, including C-

reactive protein (CRP), alpha1-antitrypsin, haptoglobin, alpha1-acid glycoprotein, and

fibrinogen. When are these proteins increased?

a) Sepsis

b) Trauma

c) Inflammation

d) Steroid use

e) HIV/AIDS

Page 14: Quiz Hematology Oncology Part 1 of 2

Hematology & Oncology – Part 1 14Mar2009

DO NOT DISTRIBUTE - 14 -

4.1) During surgery, a patient’s mean arterial pressure (MAP) drops due to blood loss.

Which of the following could the anesthesiologist administer to help the patient?

a) Antitrypsin

b) Albumin

c) Haptoglobin

d) Ceruloplasmin

e) None of the above

4.2) Which of the following functions to prevent the loss of free hemoglobin in the

kidney, conserving iron present in hemoglobin?

a) Antitrypsin

b) Albumin

c) Haptoglobin

d) Ceruloplasmin

e) Transcortin

4.3) Which of the following is a serine protease inhibitor (serpin) that can cause

emphysema if deficient as elastase is not inhibited?

a) Antitrypsin

b) Albumin

c) Haptoglobin

d) Ceruloplasmin

e) Transcortin

4.4) A teenager presents with neuropsychiatric problems including clumsiness and

behavior changes. Physical exam reveals copper Kayser-Fleischer rings in the eyes.

Which of the following would you expect to be low in plasma as it carries 90% of the

copper present?

a) Antitrypsin

b) Albumin

c) Haptoglobin

d) Ceruloplasmin

e) Transcortin

5) Which of the following is NOT true regarding iron storage and transport?

a) Hemosiderin is the partially degraded form of ferritin, containing iron

b) Ferritin is responsible for iron storage

c) Transferrin is responsible for iron transport

d) Blood ferritin levels are inversely related to transferrin levels

e) Blood ferritin levels are directly related to transferrin levels

6) Which of the following is responsible for transporting about 10% of the copper in

blood serum?

a) Antitrypsin

b) Albumin

c) Haptoglobin

d) Ceruloplasmin

e) Transferrin

7.1) A 50-year-old Caucasian male presents with progressively increasing yellowing of

the eyes (jaundice), a peculiar skin rash, and palpitations. History reveals a decreased

libido and diabetes. Physical exam reveals a bronze discoloration of the skin and

Page 15: Quiz Hematology Oncology Part 1 of 2

Hematology & Oncology – Part 1 14Mar2009

DO NOT DISTRIBUTE - 15 -

testicular atrophy. ECG shows atrial fibrillation and echo shows cardiomyopathy. A

mutation in the HFE gene is suspected and a request is made to screen first-degree

relatives. What is the diagnosis?

a) Iron deficiency anemia

b) Hemochromatosis

c) Menkes disease

d) Wilson disease

e) Diabetes insipidus

7.2) A newborn presents with seizures. Physical exam reveals “kinky” steel-colored hair

that is easily broken. Labs reveal low serum ceruloplasmin. Which of the following is

most likely?

a) Iron deficiency anemia

b) Hemochromatosis

c) Menkes disease

d) Wilson disease

e) Hepatic agenesis

8.1) Which of the following has the most positive electrophoretic mobility?

a) Alpha1 band (alpha1-acid glycoprotein, alpha1-antitrypsin)

b) Alpha2 band (alpha2-macroglobulin, haptoglobin)

c) Beta1 + Beta2 band (transferring and low-density lipoprotein)

d) Gamma band (immunoglobulins)

e) Albumin

8.2) A patient presents with multiple myeloma, a polyclonal gammopathy. A serum

protein electrophoresis is performed using a densometer (below). Which of the following

is the most abnormal?

a) Albumin

b) Alpha1 band

c) Alpha2 band

d) Beta band

e) Gamma band

8.3) Which enzyme is present

the soonest after a myocardial

infarction?

a) Myoglobin

b) CK

c) AST

d) ALT

e) LDH

8.4) Which of the following would NOT increase in serum protein electrophoresis with

trauma?

a) Albumin

b) Alpha1 band

c) Alph2 band

d) Beta band

e) All would increase

Page 16: Quiz Hematology Oncology Part 1 of 2

Hematology & Oncology – Part 1 14Mar2009

DO NOT DISTRIBUTE - 16 -

Hematology/Oncology #8 – Microbiology: Vector-Borne Infections

1) A diplomat plans to make a trip to Central America. However, just before leaving, his

doctor receives a CDC report of jungle log cutters that have fallen ill with jaundice,

instances of hemorrhage, and liver dysfunction. He insists that the diplomat be

immunized against this infectious agent before traveling to the area. Which of the

following is NOT a transmission means for this virus, yellow fever?

a) Haemagogus mosquitoes

b) Aedes aegypti mosquitoes

c) Human to human

d) Monkey to human

e) None of the above

2) For a summer trip, a woman visits the tropics for a short time. One week upon

returning, she shows signs of fever, headaches, and pain behind the eyes and in the back

and joints. She also notices a generalized rash forming. The doctor explains that the

illness will pass. This flavivirus, dengue fever, is transmitted by aedes aegypti

mosquitoes and replicates inside what kind of cells?

a) Neutrophils

b) Lymphocytes

c) Monocytes

d) Eosinophils

e) Basophils

3.1) Which of the following diseases is NOT matched correctly with the vector?

a) Rocky Mountain spotted fever; Rabbit

b) Rickettsialpox; Infected mite

c) Epidemic typhus; Louse

d) Endemic typhus; Flea

e) Scrub typhus; Mite

3.2) Which of the following diseases is NOT matched correctly with the reservoir?

a) Rocky Mountain spotted fever; Rodents

b) Rickettsialpox; Mice

c) Epidemic typhus; Human

d) Endemic typhus; Human

e) Scrub typhus; Rodent

4) A patient presents to your office complaining of fever, conjunctival redness, a severe

headache and a rash that he states started out on his wrists, ankles soles and palms and

later spread to his trunk. While on vacation in North Carolina/Oklahoma recently, the

patient found several ticks on his body. Rocky Mountain spotted fever (RMSF) is

suspected so a Weil-Felix test is ordered to find serum antigens from Proteus vulgaris. A

positive test would show latex beads coated with proteus antigens and agglutinate. Which

of the following is specific for Rickettsia species RMSF in this Weils reaction?

a) OX-19: Positive, OX-2: Positive, OX-K: Positive

b) OX-19: Positive, OX-2: Positive, OX-K: Negative

c) OX-19: Positive, OX-2: Negative, OX-K: Negative

d) OX-19: Negative, OX-2: Negative, OX-K: Positive

e) OX-19: Negative, OX-2: Negative, OX-K: Negative

Page 17: Quiz Hematology Oncology Part 1 of 2

Hematology & Oncology – Part 1 14Mar2009

DO NOT DISTRIBUTE - 17 -

5) A war veteran presents with a rash spreading outward from his trunk but sparing his

palms. He says it reminds him of what he had decades ago while in a war camp. At that

time, he had “flu-like” symptoms with the rash and recovered without treatment. Brill-

Zinsser disease is suspected. What disease is this diagnosed as?

a) Rocky Mountain spotted fever

b) Rickettsialpox

c) Epidemic typhus

d) Endemic typhus

e) Scrub typhus

6.1) A man from Ethiopia presents with relapsing fever, jaundice, mental disturbances,

bleeding, and a prolonged QT interval. There are areas of rash that the man scratches.

The clinician uses extra precautions to prevent transmission to her self and other patients.

What is the likely vector for this relapsing fever?

a) Rabbit

b) Mite

c) Louse

d) Flea

e) Tick

6.2) A camper from Colorado presents with abrupt onset of relapsing fever, rigors, and

severe headache. Exam reveals a rapid pulse. Typhoid fever is ruled out. A blood test

reveals spirochetes. What is the likely vector for this relapsing fever?

a) Rabbit

b) Mite

c) Louse

d) Flea

e) Tick

6.3) A patient is being treated with antibiotics for relapsing fever. They develop

apprehension, diaphoresis, fever, tachycardia, and hypotension. A Jarisch-Herxheimer

reaction is suspected. What drug was the patient taking?

a) Tetracycline

b) Erythromycin

c) Chloramphenicol

d) Penicillin G

e) TMP-SMX

7) A hunter from Pennsylvania presents with fever and fatigue. Physical exam reveals

erythema migrans. The clinician decides to start doxycycline instead of amoxicillin. What

is the bacteria and vector responsible?

a) Borrelia burgdorferi from ticks

b) Coxiella burnetii from goats

c) Rickettsia prowazekii from flying squirrels

d) Rickettsia rickettsia from dogs/rodents

e) Borrelia recurrentis from body louse

8.1) A patient presents with flu-like symptoms. A Geimsa-stained preparation of

peripheral blood shows morulae, where bacteria have assembled into membrane enclosed

masses. Ehrlichiosis due to E. chaffeensis is diagnosed. What cell type is infected?

a) Erythrocytes

Page 18: Quiz Hematology Oncology Part 1 of 2

Hematology & Oncology – Part 1 14Mar2009

DO NOT DISTRIBUTE - 18 -

b) Leukocytes

c) Platelets

d) CNS fluid and cells

e) Myeloid bone marrow

8.2) A patient presents with flu-like symptoms. A blood smear slows morulae and an

indirect immunofluorescence assay (IFA) is diagnostic for A. phagocytophilum

(Anaplasmosis). What cell type is infected?

a) Erythrocytes

b) Leukocytes

c) Platelets

d) CNS fluid and cells

e) Myeloid bone marrow

9.1) A man from Long Island, New York presents with high fever, shaking, and chills.

Although he has malaria-like symptoms, he denies any traveling. He says he had some

ticks removed by a friend after a hunting trip. Physical exam does not reveal erythema

migrans. A blood smear is done, shown here. The presence of ring-form inclusions and a

tetrad (Maltese cross) is noted. Which of the following is most likely?

a) Rocky Mountain spotted fever

b) Lyme disease

c) Vivax malaria

d) Falciparum malaria

e) Babesiosis

9.2) Which of the following is transmitted by

trombiculid mites ("chiggers") with the bite

site having eschar and a macular rash

following five days of illness?

a) Rickettsia typhi (Endemic typhus)

b) Rickettsia prowazekii (Epidemic typhus)

c) Rickettsia tsutsugamushi (Scrub typhus)

d) Rickettsia rickettsii (Rocky Mountain spotted fever)

e) Coxiella burnetii (Q fever)

10.1) What developmental stage of Plasmodium falciparum is shown here, noting the

“banana” shaped (”sausage”, ”boomerang”) inclusions?

a) Sporozoites

b) Liver stage

c) Merozoites

d) Asexual stage

e) Gametocytes

10.2) Symptoms associated with Plasmodium falciparum

range from fever (IL-1, TNF) to fatal renal disease and are

exclusively associated with what developmental stage?

a) Sporozoites

b) Liver stage

c) Merozoites

d) Asexual stage

e) Gametocytes

Page 19: Quiz Hematology Oncology Part 1 of 2

Hematology & Oncology – Part 1 14Mar2009

DO NOT DISTRIBUTE - 19 -

11) A Korean War veteran presents with fever, shaking, and diaphoresis every two days.

History reveals he was successfully treated for malaria while on tour with the military. A

blood smear is performed, shown here. Schüffner dots are noted. Chloroquin treatment is

begun. What was the diagnosis?

a) Plasmodium falciparum

b) Plasmodium malariae

c) Plasmodium ovale

d) Plasmodium vivax

e) Babesia (Babesiosis)

12) Which of the following is shown in the blood smear here, noting appliqué (accolé)

positioning, multiple ring forms in an individual erythrocyte, and possible Maurer dots?

a) Plasmodium falciparum

b) Plasmodium malariae

c) Plasmodium ovale

d) Plasmodium vivax

e) Babesia (Babesiosis)

13) A student reports to his college clinic complaining of “the flu.” He explains that he

has been suffering from intermittent headaches, fevers, and muscle aches. Assuming the

flu, the physician sends the student home with acetaminophen. Now, days later, the

student returns to the clinic with chills, extreme fever, and debilitating fatigue. Physical

exam reveals yellow sclera and severe splenomegaly. CBC reveals low hematocrit and

urinalysis shows hemoglobinuria. Alarmed, the doctor questions the student about recent

travels and learns that he just returned from a visit to India and says no mosquito netting

was used while sleeping. The patient is begun on mefloquine. What was the diagnosis?

a) Plasmodium falciparum

b) Plasmodium malariae

c) Plasmodium ovale

d) Plasmodium vivax

e) Babesia (Babesiosis)

14) What is the drug of choice for dormant liver vivax malaria (hypnozoite)?

a) Chloroquin

b) Quinidine

c) Mefloquine

d) Primoquin

e) Clindamycin

15) A patient with suspected plasmodium infection undergoes a blood smear, which is

negative. Inoculation of blood samples into hamsters confirms babesia (babesiosis). What

is the drug of choice?

a) Penicillin

b) Quinidine

c) Mefloquine

d) Primoquin

e) Clindamycin

16.1) An East African man is asked to leave his job after repeatedly falling asleep. He

visits the doctor hoping to cure his somnolence as well as accompanying headache and

dizziness. During the interview, the patient explains that he had suffered recurring bouts

Page 20: Quiz Hematology Oncology Part 1 of 2

Hematology & Oncology – Part 1 14Mar2009

DO NOT DISTRIBUTE - 20 -

of fever and enlarged lymph noted before the sleepiness started. The doctor decides to

perform a lumbar puncture and, after finding a flagellated protozoan in the CSF, plans to

start the patient on melarsoprol. What is the transmission vector involved (African

trypanosomiasis)?

a) Reduviid bug

b) Aedes aegypti mosquito

c) Haemagogus mosquito

d) Sand fly

e) Tsetse fly

16.2) A Mexican man complains to his doctor of worsening constipation and stomach

pains. On physical exam, the doctor is surprised to find an enlarged heart on auscultation

and moderate arrhythmia. An abdominal x-ray reveals megacolon. Head exam reveals

unilateral periorbital swelling (Romaña sign, chagoma). The clinician explains there is no

chronic treatment. What is the transmission vector involved (American trypanosomiasis)?

a) Reduviid bug

b) Aedes aegypti mosquito

c) Phlebotomous sand fly

d) Lutzomyia sand fly

e) Tsetse fly

17.1) A recent immigrant from a tropical country presents with weight loss and fever. A

physical exam reveals massive hepatosplenomegaly with associated edema as well as

hyperpigmented skin patches. The doctor orders a CBC and spleen biopsy. CBC reveals

thrombocytopenia, anemia, and leukopenia, while spleen biopsy shows macrophages

containing protozoa. The doctor begins the patient on an antimony compound. Which of

the following is most likely? (Leishmaniasis or “Kala-Azar”)

a) Leishmania donovani

b) Leishmania tropica

c) Leishmania braziliensis

17.2) A South American man presents with a cutaneous skin lesion that has ulcerated.

Physical exam reveals lesions of the mucous membranes and edema (oriental sores,

espunida). Occupational history reveals the man harvests chicle sap for gum in the

rainforest. Which of the following is most likely?

a) Leishmania donovani

b) Leishmania tropica

c) Leishmania braziliensis

17.3) What is the vector for cutaneous leishmaniasis (Leishmania tropica)?

a) Reduviid bug

b) Aedes aegypti mosquito

c) Phlebotomous sand fly

d) Lutzomyia sand fly

e) Tsetse fly

17.4) What is the vector for Leishmania donovani and Leishmania braziliensis?

a) Tsetse fly

b) Aedes aegypti mosquito

c) Phlebotomous sand fly

d) Lutzomyia sand fly

Page 21: Quiz Hematology Oncology Part 1 of 2

Hematology & Oncology – Part 1 14Mar2009

DO NOT DISTRIBUTE - 21 -

18) Which of the following is an option for drug-resistant leishmaniasis?

a) Doxycycline

b) Quinidine

c) Mefloquine

d) Sodium stibogluconate

e) Immunotherapy

19.1) A patient from a tropical village has an enormously swollen scrotum and lower

extremity. The skin around the swelling has become scaly and thick. The patient

remembers feeling enlarged nodes in the groin months before the swelling began, but

because of poor health resources in the area, he never saw a physician. Samples of his

blood drawn at night show worm-like organisms under a microscope. A visiting doctor

strongly recommends that the patient and other villagers sleep with a mosquito net to

prevent more infections. What is the helminth responsible?

a) Wuchereria bancrofti

b) Brugia malayi

c) Onchocerca volvulus

d) Loa loa filariasis

19.2) A traveling physician visits a remote riverside village in a South American country

and discovers that most of the older village inhabitants are blind. On physical exam of

some of the members, she notes skin nodules and hyperpigmented rashes. One male

villager has an extremely enlarged, hanging scrotum. To prevent other village members

from becoming blind, she administers donated ivermectin to many people in the village

and urges black fly control. What is the helminth responsible?

a) Wuchereria bancrofti

b) Brugia malayi

c) Onchocerca volvulus

d) Loa loa filariasis

19.3) A patient presents with vision problems. History reveals bouts of red itchy swelling

below the skin (Calabar swelling) of the upper extremities. Further questioning reveals a

trip to West Africa about a year ago in an area with many Chrysops (deer) and mango

flies. The eye exam is abnormal, shown. What is the helminth responsible?

a) Wuchereria bancrofti

b) Brugia malayi

c) Onchocerca volvulus

d) Loa loa filariasis

19.4) What is the vector for Brugia malayi?

a) Kissing bug

b) Mosquito

c) Mango fly

d) Sand fly

e) Tsetse fly

Hematology/Oncology #9 – Microbiology: Multisystem Zoonoses

1.1) Lymphocytic choriomeningitis (LCM) is caused by an enveloped, spherical and

segmented ss-RNA Arenavirus. What is the mode of transportation?

a) African bush rat (Mastomys natalensis)

Page 22: Quiz Hematology Oncology Part 1 of 2

Hematology & Oncology – Part 1 14Mar2009

DO NOT DISTRIBUTE - 22 -

b) Bush mouse (Calomys callosus)

c) Rodents

d) Tsetse fly

e) Unknown

1.2) A 34-year-old woman, during a visit to Nigeria, develops a fever over the course of

the 5th week of her visit. The fever progresses to headache, nausea, and diarrhea. By the

time she arrives at the hospital, her physician notes signs of pericardial effusion.

Furthermore, a diarrhea sample contains blood indicating GI hemorrhage. The physician

is quite familiar with the symptoms in that region of Nigeria. The woman is required to

remain hospitalized because she is considered contagious. How did the woman likely

contract her illness? (Lassa arenavirus)

a) African bush rat (Mastomys natalensis)

b) Bush mouse (Calomys callosus)

c) Rodents

d) Tsetse fly

e) Unknown

1.3) Marburg disease (green monkey disease) is a biosafety level-four agent and has been

weaponized for possible military used, for example by Soviet Ministry of Defense in

1990. Marburg is an enveloped ss-RNA Filovirus. Egyptian fruit bats are suspected as

one of the vectors. What is the current, most likely vector?

a) African bush rat (Mastomys natalensis)

b) Bush mouse (Calomys callosus)

c) Rodents

d) Tsetse fly

e) Unknown

1.4) An anthropologist is brought to the Emergency Room one evening for high fever,

vomiting, headache, confusion, and bloody diarrhea. He explains that he had cut an

expedition in Zaire short and returned to the U.S. when he developed the high fever.

Physical exam is remarkable for a 40ºC fever, slight hypotension, a nonpruritic rash on

the neck and arms, and a nosebleed. Upon reviewing the history, the ER physicians order

an immediate hospital quarantine of the anthropologist and his family. They then

investigate his exact itinerary in Zaire and all his close contacts in the past 3 weeks. The

next day, the patient dies with disseminated intravascular coagulation (DIC). Ebola was

suspected. What is the vector?

a) African bush rat (Mastomys natalensis)

b) Bush mouse (Calomys callosus)

c) Rodents

d) Tsetse fly

e) Unknown

1.5) What is the vector for Machupo (Bolivian hemorrhagic fever) and Junin

(Argentinian hemorrhagic fever), both Arenaviruses?

a) African bush rat (Mastomys natalensis)

b) Bush mouse (Calomys callosus)

c) Rodents

d) Tsetse fly

e) Unknown

Page 23: Quiz Hematology Oncology Part 1 of 2

Hematology & Oncology – Part 1 14Mar2009

DO NOT DISTRIBUTE - 23 -

1.6) Hantaan is an enveloped 3-RNA fragment virus that causes Korean hemorrhagic

fever. An infected host (e.g. mouse, rat) transmits infection to blood sucking arthropod

where the virus replicates and transmits to a new host. What type of virus is involved?

a) Filovirus

b) Arenavirus

c) Bunyavirus

d) Paramyxovirus

e) Orthomyxovirus

2) A cattle farmer goes to his doctor complaining of a mild cough and fever. He says that

the fever began abruptly several days ago. His occupation as a livestock slaughterer leads

the doctor towards a diagnosis, and tetracycline is administered. Diagnosis is confirmed

via a negative Weil-Felix test. What was the diagnosis?

a) Rickettsia typhi (Endemic typhus)

b) Rickettsia prowazekii (Epidemic typhus)

c) Rickettsia tsutsugamushi (Scrub typhus)

d) Rickettsia rickettsii (Rocky Mountain spotted fever)

e) Coxiella burnetii (Q fever)

3.1) A man from Hawaii comes to the Emergency Room with a 1-week history of flu-like

symptoms with photophobia. His severe headache, cough, and myalgias suggest to the

physician some kind of respiratory infection. However, more careful physical exam

reveals conjunctival suffusion and macular rash (Weil syndrome). Lab findings include

elevated serum bilirubin, alkaline phosphatase, aminotransferases, and creatine

phosphokinase. With this clinical picture and lab results, the physician prescribes

penicillin G immediately. The physician suspects the man got this disease while

swimming in water contaminated with rat urine. His suspicions are confirmed later when

a question mark shaped spirochete is isolated from the patient’s blood. What was the

diagnosis?

a) Streptobacillus moniliformis or Spirillum minus (minor)

b) Borrelia burgdorferi or Borrelia recurrentis

c) Leptospira interrogans

d) Treponema pallidum

e) Mycobacterium leprae

3.2) Rat Bite fever is diagnosed either by microscopy from a wound site, lymph nodes,

and blood, or via dark field microscopy. Which of the following is most likely?

a) Streptobacillus moniliformis or Spirillum minus (minor)

b) Borrelia burgdorferi or Borrelia recurrentis

c) Leptospira interrogans

d) Treponema pallidum

e) Mycobacterium leprae

4) A doctor is struggling to diagnose a woman’s flulike illness. She complains of a fever

that rises during the day and peaks after dinner, fatigue, spinal tenderness, and loss of

appetite. The fever is associated with profuse sweating. Her lymph nodes are enlarged in

physical exam. The doctor has trouble narrowing down the possible etiologies until he

hears that the woman tasted goat cheese at a local French village a month before the onset

of her symptoms. Which of the following is most likely?

a) Brucella abortus

Page 24: Quiz Hematology Oncology Part 1 of 2

Hematology & Oncology – Part 1 14Mar2009

DO NOT DISTRIBUTE - 24 -

b) Brucella canis

c) Brucella melitensis

d) Brucella suis

e) Francisella tularensis

5) Which of the following is NOT true of the diagnosis and treatment for Brucellosis

(Malta Fever)?

a) IgM antibodies are present in acute brucellosis

b) IgG and IgA antibodies are present in chronic brucellosis

c) Tetracycline is a treatment option of choice

d) Streptomycin is a treatment option of choice

e) TMP-SMX should be avoided due to adverse side effects

6) A woman from Arkansas presents to the doctor with a small but persistent black ulcer

on her arm. The area near the ulcer is erythematous and tender. Her axillary lymph nodes

on the same side are enlarged. She believes the ulcer may be related to a tick bite that

occurred on her arm while tending to her rabbit farm. A lab test shows growth on BCYE

agar. Which of the following is most likely?

a) Pasteurella multocida

b) Borrelia burgdorferi

c) Francisella tularensis

d) Bartonella quintana

e) Bartonella henselae

7) What subspecies of Francisella tularensis is the most common in the United States?

a) Tularensis (Type A)

b) Holarctica (Type B)

c) Mediasiatica

d) Novicida

8) What is the treatment of choice for Francisella tularensis?

a) TMP-SMX

b) Amphotericin B

c) Fluoroquinolones

d) Streptomycin

e) Penicillin

9.1) A homeless injection drug user from Seattle presents with a relapsing five-day fever.

Examination finds several body lice. The man complains of fever, severe soreness and

pain of the leg bones, and pain when moving his eyes. The clinician suspects a disease

usually seen during war. Lab results including a Weil-Felix test are diagnostic and

gentimycin treatment is begun. Which of the following is most likely?

a) Pasteurella multocida

b) Borrelia burgdorferi

c) Francisella tularensis

d) Bartonella quintana

e) Bartonella henselae

9.2) A homeless man presents to the Emergency Room days after being scratched by a

feral (stray) cat. The man has a high fever and lymphadenopathy. Further examination

reveals peliosis hepatis. The physician orders an HIV test and does not begin antibiotic

treatment, as there will likely be no response. Which of the following is most likely?

Page 25: Quiz Hematology Oncology Part 1 of 2

Hematology & Oncology – Part 1 14Mar2009

DO NOT DISTRIBUTE - 25 -

a) Pasteurella multocida

b) Borrelia burgdorferi

c) Francisella tularensis

d) Bartonella quintana

e) Bartonella henselae

10) Bacillary angiomatosis is seen in immunocomromised patients as well as sub-acute

endocarditis, associated with which of the following?

a) Francisella

b) Bartonella

c) Pasteurella

d) Brucella

e) Leptospira

Match the infective organism with the vector:

11.1) R. akari a) Tick

11.2) R. typhi b) Louse

11.3) E. chafeensis c) Flea

11.4) A. phagocytophilum d) Mite

11.5) R. tsutsugamushi

11.6) R. prowazekii

Hematology/Oncology #10 – Microbiology: Epstein-Barr Virus & Cytomegalovirus

1.1) A 20-year-old female college student reports to the medical center complaining of

“the flu.” She reports fever, night sweats, a very painful sore throat, and headaches. She

thought she could endure the illness, but she became frustrated after feeling “so sleeping

all the time.” Physical exam reveals enlarged lymph nodes and a slight splenomegaly.

Results from a blood smear later that day reveal lymphocytosis with about 20%

lymphocytes having an abnormally large nucleus and vacuolated cytoplasm. The student

is assured that the illness will spontaneously resolve within 2-3 weeks, but that she

should avoid contact sports during that time. She is told not to share drinks, to prevent

spread of the illness. What is the most likely way she received this illness?

a) Semen

b) Blood

c) Saliva

d) Urine

e) Breast milk

1.2) A 34-year-old kidney transplant patient currently on immunosuppresants complains

of shortness of breath and coughing. Physical exam reveals fever and abnormal lung

sounds while chest x-ray indicates interstitial infiltrates in the lungs. No cysts are

detected on silver stain on bronchoalveolar lavage fluid, ruling out Pneumocystis jejuni

infection. The doctor makes a diagnosis after viewing a sample of the patient’s lung

tissue, which shows abnormal giant cells with intranuclear inclusions that show a clearing

halo around the nucleus. Which of the following is most likely?

a) Herpesvirus 1 (HSV-1)

b) Herpesvirus 2 (HSV-2)

c) Herpesvirus 3 (VZV)

d) Herpesvirus 4 (EBV)

Page 26: Quiz Hematology Oncology Part 1 of 2

Hematology & Oncology – Part 1 14Mar2009

DO NOT DISTRIBUTE - 26 -

e) Herpesvirus 5 (CMV)

2) Which of the following is indicative of a current and previous Epstein-Barr virus

(EBV, mononucleosis) infection?

a) IgM antibody

b) VCA antibody

c) EBNA antibody

d) IgM and VCA

e) VCA and EBNA

3) Which of the following is responsible for mononucleosis?

a) Herpesvirus 1 (HSV-1)

b) Herpesvirus 2 (HSV-2)

c) Herpesvirus 3 (VZV)

d) Herpesvirus 4 (EBV)

e) Herpesvirus 5 (CMV)

4.1) What is shown here?

a) Atypical lymphocyte (Downey cell)

b) Positive heterophile antibody

c) Owl’s eye inclusion body

d) Burkitt lymphoma

e) Nothing abnormal

4.2) What is shown here?

a) Atypical lymphocyte (Downey cell)

b) Positive heterophile antibody

c) Owl’s eye inclusion body

d) Burkitt lymphoma

e) Nothing abnormal

5) Which of the following is associated with cytomegalovirus (CMV), not EBV?

a) Atypical lymphocyte (Downey cell)

b) Positive heterophile antibody

c) Owl’s eye inclusion body

d) Burkitt lymphoma

6) Burkitt lymphoma is a co-infection associated with EBV and malaria. Which of the

following describes the likely demographic?

a) Elderly in Africa

b) Children in Africa

c) Cave spelunkers in Ohio

d) Stagnant water swimmers in South America

e) River village members in West Africa

7) A young adult male presents to the clinic complaining of problems with his gums and

skin. Upon examination of the dark raised lesions, it is clear the man has Kaposi sarcoma

(HHV8). Flow cytometry for the patient is shown. Which of the following is most likely?

a) AIDS

b) Steroid use

c) Latent EBV

d) Lymphoma

e) Multiple sclerosis

Page 27: Quiz Hematology Oncology Part 1 of 2

Hematology & Oncology – Part 1 14Mar2009

DO NOT DISTRIBUTE - 27 -

Hematology/Oncology #11 – Immunology: Secondary Immunodeficiency

1) The HIV envelope, made of gp120 molecules, binds CD4 in order to enter target cells.

The CCR5 chemokine receptor on macrophages and dendritic cells is used by HIV to

gain entry and start infection after which of the following?

a) Blood transfusion

b) Solid organ transplant

c) Sexual contact

d) IV drug use

e) Handshaking

2) During the seroconversion illness stage of HIV infection, which of the following

would NOT be seen?

a) Prolonged fever (4 – 14 days) and aching limbs

b) Red blotchy rash over the trunk

c) Ulceration in the mouth or genitals

d) Aversion to light and severe headaches

e) Low CD4+ cell count

3) Which of the following opportunistic infections seen in AIDS patients occurs when

CD4+ T-cell counts are below 50cells/microliter?

a) Non-Hodgkin lymphoma

b) Kaposi sarcoma (HHV8)

c) Candida albicans (Candidiasis)

d) M. avium intercellular (MAC)

e) Cytomegalovirus (CMV)

4) Which of the following would be descriptive of a CD4+ versus CD8+ flow cytometry

graph of an AIDS patient?

a) High CD8+, High CD4+

b) High CD8+, Low CD4+

c) Low CD8+, High CD4+

d) Low CD8+, High CD4+

5) Which of the following would be likely if a patient were infected with HIV that had a

deleted nef gene?

a) The patient would die very quickly

b) The patient’s immune system could recognize HIV

c) The virus could not replicate within CD4+ cells

d) The virus would remain dormant in lymph nodes

e) The viral load would increase rapidly

6) Which of the following is true for patients who are homozygous for a polymorphism in

the CCR5 receptor?

a) They have a decreased risk of HIV infection after exposure

b) They are unable to contract HIV from non-sexual contact

c) They will be highly contagious at the onset of AIDS

d) They will transition from HIV to AIDS very quickly

e) They have a greatly increased risk of HIV infection after exposure

Page 28: Quiz Hematology Oncology Part 1 of 2

Hematology & Oncology – Part 1 14Mar2009

DO NOT DISTRIBUTE - 28 -

7) A female prostitute has enhanced cytotoxic T-lymphocyte (CTL) response to HIV. She

has had unprotected intercourse many times over the past year and has probably come in

contact with HIV. Which of the following is likely?

a) She will develop AIDS within a year

b) She will have no trace of HIV in her blood

c) If she is exposed to HIV again, she will progress to AIDS

d) She will remain well for several years

e) If she is exposed to HIV again, her CD4+ count will decline to near zero

Hematology/Oncology #12 – Immunology & Microbiology: Transplantation

1) Which of the following is a transplant between members of the same species but who

are not identical twins?

a) Autologous transplant

b) Syngeneic transplant

c) Allogeneic transplant

d) Xenogeneic tranplant

2) A transplantation is indicated if there is irreversible damage to an organ and there is no

major transplantation risk. Patients with Goodpasteur syndrome, due to antibodies,

should not receive what type of transplant?

a) Corneal

b) Blood

c) Liver

d) Kidney

e) Pancreas

3) Which of the following types of drugs is the most commonly used in transplantation to

minimize rejection, and is used in both low doses to prevent early rejection and high

doses to treat episodes of rejection?

a) Corticosteroids

b) T-cell signal inhibitors

c) Anti-proliferative drugs

d) Monoclonal antibodies

e) Antibiotics and antivirals

4.1) What phase of graft rejection involves HLA class I incompatibility or minor

histocompatibility antigens?

a) Hyperacute

b) Acute

c) Chronic

4.2) What type of hypersensitivity reaction is seen in hyperacute graft rejection?

a) Type I

b) Type II

c) Type III

d) Type IV

4.3) What type of hypersensitivity reaction is seen in acute graft rejection?

a) Type I

b) Type II

c) Type III

Page 29: Quiz Hematology Oncology Part 1 of 2

Hematology & Oncology – Part 1 14Mar2009

DO NOT DISTRIBUTE - 29 -

d) Type IV

5) Which of the following is NOT true of minor histocompatibility antigens?

a) They are detected with standard tissue typing techniques before transplantation

b) They have different amino acid sequences in the donor and recipient

c) They are encoded by genes situated outside of the HLA

d) These antigens can cause rejection even with an HLA match

6) What is the chance that siblings would be a “perfect match” in that they had the same

human leukocyte antigens (HLA)?

a) 100%

b) 75%

c) 50%

d) 25%

e) 0%

7) Following a stem cell transplant (SCT), myeloid cells are regenerated from

pleuripotent stem cells from bone marrow, peripheral stem cells, or cord blood. Which of

the following would be an indication for SCT?

a) Non-Hodkin lymphoma

b) Stroke or myocardial infarction

c) Hematological malignancy

d) Malaria or other blood disease

e) Secondary immunodeficiency (e.g. AIDS)

8) Which of the following describes donor treatment prior to stem cell transplantation?

a) IV injection of corticosteroids, antiviral agents, and antibacterial agents

b) High-dose chemotherapy or radiotherapy to destroy host stem cells

c) Whole blood infusion with CD4+ cell enhancement

d) Reduction in fluids and blood pressure to accommodate stem cells

e) No preparation is necessary

9) When does graft versus host disease (GVHD) occur?

a) ABO blood type mismatch

b) HLA type mismatch

c) Major histocompatibility antigen mismatch

d) Minor histocompatibility antigen mismatch

e) A or B

f) C or D

10) When cross-matching ABO blood types, what test is used to determine when cells are

coated with antibody?

a) Direct Coombs test

b) Indirect Coombs test

c) Positive heterophile test

d) Anti-human IgG antibody test

e) HLA cross matching test

11.1) What type of hypersensitivity reaction is involved in an ABO blood type mismatch?

a) Type I

b) Type II

c) Type III

d) Type IV

Page 30: Quiz Hematology Oncology Part 1 of 2

Hematology & Oncology – Part 1 14Mar2009

DO NOT DISTRIBUTE - 30 -

11.2) Alloimmune hemolysis, which most frequently occurs in pregnancy, involves the

rhesus (Rh) antigen with exposure causing a production of which immunoglobulin?

a) IgA

b) IgD

c) IgE

d) IgG

e) IgM

12) Which of the following is NOT a mechanism by which the maternal immune system

can tolerate a fetus?

a) The placenta secretes cytokines, skewing the maternal response toward TH2

b) The trophoblast secretes low levels of HLA class I, preventing recognition

c) Estrogen, a steroid hormone, is secreted continuously

d) The trophoblast secretes high levels of HLA-G, inhibiting NK cells

e) The placenta secretes hormones to inhibit IgG and IgA during pregnancy

13) Which of the following opportunistic infections seen in transplant patients would

NOT occur within the first few weeks after transplantation and immunosuppression?

a) Nosocomial pneumonia

b) Herpes simplex virus

c) Hepatitis B

d) C. difficile colitis

e) Urinary tract infections

Hematology/Oncology #13 – Pharmacology: Blood, Inflammation, & Gout

1) Which of the following is NOT used in anemia?

a) Iron

b) Vitamin B12

c) Folic acid

d) Vitamin K

e) Erythropoietin

2) Large amounts of which of the following leads to gastroenteritis with vomiting, pain,

and diarrhea, and is not bound well with activated charcoal, but can be bound with

deferoxamine?

a) Vitamin B12

b) Iron

c) Folic acid

3) Which of the following requires intrinsic factor to bind and be absorbed?

a) Vitamin B12

b) Iron

c) Folic acid

4) Which of the following is NOT a hematopoietic growth factor?

a) Erythropoietin

b) Interleukin 1

c) Interleukin 11

d) Granulocyte colony stimulating factor (G-CSF)

e) Granulocyte macrophage colony stimulating factor (GM-CSF)

Page 31: Quiz Hematology Oncology Part 1 of 2

Hematology & Oncology – Part 1 14Mar2009

DO NOT DISTRIBUTE - 31 -

5) Which of the following has side-effects producing fever, malaise, arthralgies,

myalgias, and capillary leak syndrome with peripheral edema and pericardial effusions?

a) Erythropoietin

b) Myeloid growth factor G-CSF

c) Myeloid growth factor GM-CSF

d) Megakaryocyte growth factor IL-11

e) None of the above

6) Which of the following has side-effects producing fatigue, headache, dizziness, and

cardiovascular effects from the increase in renal resorption of sodium?

a) Erythropoietin

b) Myeloid growth factor G-CSF

c) Myeloid growth factor GM-CSF

d) Megakaryocyte growth factor IL-11

e) None of the above

Hematology/Oncology #14 – Pharmacology: Coagulation

1) Which of the following is affected by thrombolytic agents and aminocaproic acid?

a) Protein C & S

b) Clotting factor II

c) Clotting factor VII

d) Plasminogen

e) Clotting factor IX

f) Clotting factor X

2) Heparin potentiates the action of antithrombin III and affects, at the end, what factor?

a) Clotting factor II

b) Clotting factor VII

c) Clotting factor IX

d) Clotting factor X

e) Clotting factor XI

2.2) Low molecular weight heparin (LMWH) affects what factor (end factor affected)?

a) Clotting factor II

b) Clotting factor VII

c) Clotting factor IX

d) Clotting factor X

e) Clotting factor XI

3) Hirudin is the most potent natural inhibitor of thrombin. Enhanced effect may be seen

due to accumulation in patients with what organ dysfunction?

a) GI

b) Spleen

c) Liver

d) Kidneys

e) Adrenals

4) Warfarin (Coumadin), is contraindicated in which of the following cases?

a) Diabetes

b) Pregnancy

c) Recent stroke

Page 32: Quiz Hematology Oncology Part 1 of 2

Hematology & Oncology – Part 1 14Mar2009

DO NOT DISTRIBUTE - 32 -

d) Recent MI

e) Endocarditis

5) Which of the following drugs, which all interact with anticoagulants, binds warfarin in

the intestine affecting its absorption and bioavailability?

a) Phenylbutazone

b) Sulfinpyrazone

c) Aspirin

d) Barbiturates/Rifampin

e) Cholestyramine

6) Which of the following body factors or drugs taken with oral anticoagulants would

lead to decreased prothrombin time?

a) Hypothyroidism

b) Cephalosporins

c) Hepatic disease

d) Cimetidine

e) Amiodarone

7) Which of the following is considered an anti-platelet agent?

a) Streptokinase

b) Alteplase

c) Clopidogrel

d) Reteplase

e) Tissue plasminogen activator (tPA)

8) What is the mechanism of action for clopidogrel (and ticlopidine)?

a) Inhibition of prostaglandin metabolism

b) Blockade of GP IIb/IIIa receptors on platelets

c) Catalyzing the formation of serum protease plasmin

d) Inhibition of ADP induced platelet aggregation

e) Activation of the plasminogen zymogen

9) Which of the following has side-effects including allergic reaction, bloody

vomit/stools, severe headache, confusion, unexplained bleeding and neutropenia (a

reason why another drug is preferred)?

a) Aspirin

b) Streptokinase

c) Ticlopidine

d) tPa

e) Glycoprotein IIb/IIIa inhibitors

10) Which of the following drugs is NOT used to prevent clotting?

a) Warfarin

b) LMWH

c) Heparin

d) Clopidogrel

11) Which of the following drugs used in bleeding is NOT matched correctly with its

affected clotting factors?

a) Vitamin K: VII, IX, X

b) Plasma fractions: VIII, IX

c) Desmopressin acetate (arginine vasopressin): VIII

Page 33: Quiz Hematology Oncology Part 1 of 2

Hematology & Oncology – Part 1 14Mar2009

DO NOT DISTRIBUTE - 33 -

d) Cryoprecipitate: II

12) A patient presents with hemophilia B (Christmas disease). What drug should they be

given?

a) Plasma fibrinogen and cryoprecipitate

b) Plasma, factor IX-prothrombin complex

c) Fresh-frozen plasma

d) Treatment not needed

e) Concentrate (AT III-specific)

13) Which of the following is NOT contained in cryoprecipitate (Cryoprecipitated

Antihemophilic Factor)?

a) Fibronectin

b) Factor XIII

c) vWF

d) Factor VII

e) Factor X

14) Which of the following is a synthetic inhibitor of fibrinolysis, competitively

inhibiting plasminogen activation?

a) Aminocaproic acid (Amicar)

b) Aprotinin (Translol)

c) Clopidogrel (Plavix)

d) Urokinase

e) tPA

15) Which of the following is a serine protease inhibitor that inhibits fibrinolysis by free

plasmin and also inhibits the plasmin-streptokinase complex?

a) Aminocaproic acid (Amicar)

b) Aprotinin (Translol)

c) Clopidogrel (Plavix)

d) Urokinase

e) tPA

Hematology/Oncology #15 – Pharmacology: Anti-Malarial Agents

1) Which of the following drugs used in the treatment of malaria affects the

exoerythrocytic cycle, not the erythrocyctic cycle?

a) Chloroquine

b) Primaquine

c) Quinine

d) Mefloquine

e) Pyrimethamine

f) Proguanil

g) Atovaquone

2) What is the drug of choice for infections caused by P. vivax or P. falciparum?

a) Chloroquine

b) Primaquine

c) Quinine

d) Mefloquine

e) Pyrimethamine

Page 34: Quiz Hematology Oncology Part 1 of 2

Hematology & Oncology – Part 1 14Mar2009

DO NOT DISTRIBUTE - 34 -

3) What is the drug of choice for malarial infections that are resistant to chloroquine,

which is also combined with tetracycline, clindamycin or Fansidar?

a) Proguanil

b) Primaquine

c) Quinine

d) Mefloquine

e) Pyrimethamine

4) What is the drug of choice for prevention of relapse of malaria, as it is highly active

against the hepatic stages of P. vivax?

a) Chloroquine

b) Primaquine

c) Quinine

d) Mefloquine

e) Pyrimethamine

5) What is the drug of choice for malarial prophylaxis in regions that are chloroquine

resistant (which involves the membrane P-glycoprotein pump)?

a) Atovaquone

b) Primaquine

c) Quinine

d) Mefloquine

e) Pyrimethamine

6) Which of the following would decrease the oral administration of chloroquine?

a) TMP-SMX

b) Amiodarone

c) Vegan diet

d) Meat products

e) Antacids

7) Chloroquine should be used with caution in patients with what type of disease?

a) CNS

b) GI

c) Renal

d) Hepatic

e) Splenic

8) What is the most serious and frequent adverse effect seen in primaquine, especially in

patients with G6PD deficiency?

a) Thrombocytopenia

b) Thrombophilia

c) Excess bleeding

d) Hemolysis

e) Cinchonism (tinnitus, headache, nausea)

9) What of the most common side-effect seen in patients taking quinine?

a) Thrombocytopenia

b) Atrial fibrillation

c) Hypotension

d) Hemolysis and black fever

e) Cinchonism (tinnitus, headache, nausea)

Page 35: Quiz Hematology Oncology Part 1 of 2

Hematology & Oncology – Part 1 14Mar2009

DO NOT DISTRIBUTE - 35 -

10) Which of the following drugs has a prolonged half-life (1-4 weeks), should not be

given to patients with epilepsy or psychiatric disorders, and can cause adverse reactions

that may be indistinguishable from malaria symptoms?

a) Atovaquone

b) Primaquine

c) Quinine

d) Mefloquine

e) Pyrimethamine

11) Which of the following is used for chloroquine-resistant P. falciparum, is the drug of

choice for acute attacked as an alternative to mefloquine and doxycycline for

prophylaxis, and acts by inhibiting dihydrofolate reductase?

a) Atovaquone

b) Primaquine

c) Quinine

d) Proguanil

e) Pyrimethamine

12) Which of the following is used for multidrug resistant P. falciparum?

a) Atovaquone

b) Primaquine

c) Pyrimethamine

d) Proguanil

e) Halofantrine

Hematology/Oncology #16 – Pharmacology: Gout Therapeutics

1) Along with uricosurics, what is the drug of choice in chronic gout?

a) NSAIDs

b) Colchicine

c) Allopurinol

d) Glucocorticoids

e) Analgesics

2) Gout may be precipitated by trauma, infection, or surgery and is due to the buildup of

uric acid deposits (trophi). Lab samples show negatively-birefringent “needles” and may

show hyperuricemia. What are of the body is usually the first affected?

a) First metatarsal

b) Knees

c) Elbows

d) Low back

e) Face

3) Which of the following drugs decreases uric acid production (versus increased urate

excretion)?

a) Probenecid (Benemid)

b) Allopuranol (Zyloprim)

c) Sulfinpyrazone (Anturane)

d) None of the above

4) A patient presents with gout that is not relived by ibuprofen. What NSAID would be

recommended?

Page 36: Quiz Hematology Oncology Part 1 of 2

Hematology & Oncology – Part 1 14Mar2009

DO NOT DISTRIBUTE - 36 -

a) Naproxen

b) Aspirin

c) Indomethacin

d) Meloxicam

5) What is the major side effect seen in colchicine?

a) Nausea

b) Diarrhea

c) Bleeding

d) Optic neuritis

e) Ototoxicity

Hematology/Oncology #17 – Pharmacology: Anticoagulants

1) Which of the following is NOT true?

a) Platelet aggregation is mostly associated with arterial thrombi

b) Coagulation is mostly associated with venous thrombi

c) The intrinsic pathway is activated by surface contact with a foreign body

d) The extrinsic pathway is activated by tissue factors such as VII

2) Lepirudin (leech protein), antithrombin III, and warfarin are categorized as:

a) Thrombolytics

b) Anticoagulants

c) Antiplatelet drugs

d) Replacement factors

e) Plasminogen inhibitors

3) Which of the following is a clinical use of antiplatelets?

a) Myocardial infarction

b) Ischemic stroke

c) Deep vein thrombosis

d) Arterial thrombosis

e) Arterial disease

4) Which of the following is given as an oral anticoagulant, along with anisindione and

dicumarol?

a) Heparin

b) Lepirudin

c) Warfarin

d) Antithrombin III

e) Enoxaparin

5) Which of the following drugs potentiates the action of antithrombin III?

a) Heparin

b) Lepirudin

c) Warfarin

d) Hirudin

e) Dalteparin

6) The pentasaccharide sequence in heparin binds to which of the following?

a) Thrombin

b) Antithrombin

c) Factor Xa

Page 37: Quiz Hematology Oncology Part 1 of 2

Hematology & Oncology – Part 1 14Mar2009

DO NOT DISTRIBUTE - 37 -

d) Factor VIIa

e) Factor IXa

7) What form of heparin has a 2-hour delay of onset and is not associated with

unpredictable absorption and local bleeding?

a) Oral

b) IV

c) IM

d) SubQ

8) A patient is taking LMWH (enoxaparin or dalteparin) and complains of leg pain. A

CBC and differential taken during treatment shows a 50% drop in platelets (down

100,000/mcL). The clinician fears immunologic mediated heparin-induced

thrombocytopenia (HIT). Heparin is stopped, but anticoagulation must continue. What

drug should be given?

a) Warfarin

b) Dalteparin

c) Vitamin K

d) Aspirin

e) Lepirudin

9) Drugs that inhibit platelet function and increase bleeding are contraindicated in HIT.

This includes:

a) Warfarin

b) Dalteparin

c) Vitamin K

d) Aspirin

e) Lepirudin

10) What drug can neutralize heparin in the case of serious hemorrhage?

a) Protamine

b) Warfarin

c) Clopidogrel

d) Urokinase

e) Aminocaproic acid

11) Warfarin (Coumadin) is a commonly prescribed drug for embolism risk, such as in

patients with atrial fibrillation. If a patient takes too many pills and presents with

bleeding, which of the following should be given?

a) Warfarin

b) Dalteparin

c) Vitamin K

d) Aspirin

e) Lepirudin

12) Which of the following is a powerful and specific thrombin inhibitor made from the

saliva of the leech, accumulates in renal insufficiency, and in some patients may lead to

the development of antibody directed against the thrombin-drug complex?

a) Heparin

b) Protamine

c) Warfarin

d) Hirudin

Page 38: Quiz Hematology Oncology Part 1 of 2

Hematology & Oncology – Part 1 14Mar2009

DO NOT DISTRIBUTE - 38 -

e) Dalteparin

13) Which of the following cytochrome P-450 drugs would reduce the clearance and

increased the efficacy of a given dose of warfarin?

a) Phenytoin

b) Cimetidine

c) Rifampin

d) Barbiturates

e) Carbamazepine

14) Anisindione has great toxicity and is reserved for patients who are unresponsive to:

a) Heparin

b) Protamine

c) Warfarin

d) Hirudin

e) Dalteparin

15) Aside from bleeding, what other side-effect of dicumarol has led to its lack of use in

the United States?

a) Thrombosis

b) Hepatotoxicity

c) Nephrotoxicity

d) Vasocontriction

e) Birth defects

Hematology/Oncology #18 – Pharmacology: Anti-Platelet Agents

1) Which of the following is a phosphodiesterase III (PDE-III) inhibitor?

a) Acetylsalicylic acid (Aspirin)

b) Clopidogrel (Plavix)

c) Cilostazol (Pletal)

d) Abciximab (Rheopro)

e) Dipyridimole (Persantine)

2) Which of the following is an adenosine reuptake inhibitor?

a) Acetylsalicylic acid (Aspirin)

b) Clopidogrel (Plavix)

c) Cilostazol (Pletal)

d) Abciximab (Rheopro)

e) Dipyridimole (Persantine)

3) Which of the following is a glycoprotein IIb/IIIa (GP IIb/IIIa) inhibitor?

a) Acetylsalicylic acid (Aspirin)

b) Clopidogrel (Plavix)

c) Cilostazol (Pletal)

d) Abciximab (Rheopro)

e) Dipyridimole (Persantine)

4) Which of the following irreversibly inhibits prostaglandin H synthase?

a) Acetylsalicylic acid (Aspirin)

b) Clopidogrel (Plavix)

c) Cilostazol (Pletal)

d) Abciximab (Rheopro)

Page 39: Quiz Hematology Oncology Part 1 of 2

Hematology & Oncology – Part 1 14Mar2009

DO NOT DISTRIBUTE - 39 -

e) Dipyridimole (Persantine)

5) Which of the following inhibits ADP-induced platelet fibrinogen binding and further

platelet-platelet interactions?

a) Clopidogrel (Plavix)

b) Ticlopidine (Ticlid)

c) Tirofiban (Aggrastat)

d) Eptifibatide (Integrilin)

e) Extended-release dipyridamole with aspirin (Aggrenox)

6) Which of the following inhibits cyclooxygenase-1 (COX blocker)?

a) Acetylsalicylic acid (Aspirin)

b) Clopidogrel (Plavix)

c) Tirofiban (Aggrastat)

d) Eptifibatide (Integrilin)

e) Dipyridimole (Persantine)

7) Aspirin has dose-related side effects relating to what system?

a) GI

b) Kidney

c) Liver

d) Spleen

e) CNS

8) What side effect is seen with ticlopidine but not clopidogrel?

a) Bloody vomit/diarrhea

b) Neutropenia

c) Allergic reaction

d) Severe headache, confusion, dizziness

e) Unexplained bruising and bloody nose

9) Which of the following produces an immediate and profound inhibition of platelet

activity (80% reduction), lasts for 12-36 hours after infusion termination, and is used

after percutaneous coronary angioplasty?

a) Eptifibatide (Integrilin)

b) Clopidogrel (Plavix)

c) Cilostazol (Pletal)

d) Abciximab (Rheopro)

e) Dipyridimole (Persantine)

10) Which of the following is associated with bleeding, nausea, fever, headache, rash,

and thrombocytopenia?

a) Eptifibatide (Integrilin)

b) Clopidogrel (Plavix)

c) Cilostazol (Pletal)

d) Abciximab (Rheopro)

e) Dipyridimole (Persantine)

11) Which of the following is contraindicated in severe coronary artery disease (CAD) as

well as subvalvular aortic stenosis?

a) Eptifibatide (Integrilin)

b) Clopidogrel (Plavix)

c) Cilostazol (Pletal)

Page 40: Quiz Hematology Oncology Part 1 of 2

Hematology & Oncology – Part 1 14Mar2009

DO NOT DISTRIBUTE - 40 -

d) Abciximab (Rheopro)

e) Dipyridimole (Persantine)

12) Which of the following is recommended as the first-line antiplatelet drug and has

been shown to reduce the risk of serious vascular events in high-risk patients?

a) Acetylsalicylic acid (Aspirin)

b) Clopidogrel (Plavix)

c) Cilostazol (Pletal)

d) Abciximab (Rheopro)

e) Dipyridimole (Persantine)

13) Which of the following has been shown to induce systemic inflammatory response

syndrome (SIRS), characterized by cellulitis-like rash, fever, chills, tachycardia,

hypotension, leukopenia, and impaired liver function?

a) Acetylsalicylic acid (Aspirin)

b) Clopidogrel (Plavix)

c) Cilostazol (Pletal)

d) Abciximab (Rheopro)

e) Dipyridimole (Persantine)

Hematology/Oncology #19 – Pathology

1) Which of the following would be seen a few days after anemia of acute blood loss?

a) Thrombocytopenia

b) Leukopenia

c) Leukocytosis

d) Reticulocytosis

e) Thrombocytosis

2) Which of the following is often seen in patients with hemolytic anemia, which is

defined as shortened RBC life span (less than 120 days)?

a) Spur cells (acanthocytes)

b) Burr cells (echinocytes)

c) Heinz bodies

d) Howell-Jolly bodies

e) Schistocytes

3) A bongo drummer presents with jaundice. Lab results show decreased haptoglobin,

hemoglobinemia, and hemoglobinuria. Which of the following is most likely?

a) Intravascular hemolysis

b) Extravascular hemolysis

c) Spherocytosis

d) Sickle cell anemia

e) Thalassemia

4) A patient presents with jaundice. Testing reveals anemia, decreased haptoglobin and

no signs of hemoglobinemia or hemoglobinuria. If extravascular hemolysis is suspected,

what is the major organ involved?

a) Liver

b) Spleen

c) Pancreas

d) Heart

Page 41: Quiz Hematology Oncology Part 1 of 2

Hematology & Oncology – Part 1 14Mar2009

DO NOT DISTRIBUTE - 41 -

e) Kidney

5) A patient presents with suspected anemia. Blood tests show a hematocrit of 25% and a

mean corpuscle (cell) volume of 90fL. Osmotic fragility test is positive. A defect in

spectrin (and ankyrin) is suspected. Which of the following is most likely?

a) Glucose-6-phosphate dehydrogenase deficiency

b) Autoimmune hemolytic anemia

c) Spherocytosis

d) Sickle cell anemia

e) Thalassemia

6) An African American presents with hemolytic crisis. They report eating many broad

beans and are being treated for a malaria infection received on a recent mission trip.

Blood tests show Heinz bodies and bite cells. Which of the following is most likely?

a) Glucose-6-phosphate dehydrogenase deficiency

b) Autoimmune hemolytic anemia

c) Megaloblastic anemia

d) Sickle cell anemia

e) Pernicious anemia

7) What change is seen at the 6th position of the beta-globin chain in patients with sickle

hemoglobin (HbS)?

a) Valine for glycine

b) Glycine for valine

c) Valine for glutamic acid

d) Glutamic acid for valine

e) None of the above

8) An African American child presents with pain in the joints and coughing. Physical

exam reveals splenomegaly. Pneumonia is diagnosed after lab cultures show the

encapsulated bacteria Strep. pneumoniae. Osteomyelitis is also found. What is the most

common bacteria responsible for the osteomyelitis in this patient?

a) Staph. aureus

b) Strep. pneumoniae

c) H. influenzae

d) Salmonella

e) Enterobacter

9) A child presents with splenomegaly. A blood test shows a hematocrit of 20%,

hemoglobin of 7g/dL, and a mean corpuscle volume (MCV) of 45fL. An iron profile

returns normal. A stool guaiac test is negative. Which of the following is most likely?

a) Gastrointestinal cancer

b) Hemolytic anemia

c) Paroxysmal nocturnal hemoglobinuria

d) Hereditary spherocytosis

e) Thalassemia

10) A leukemia patient presents with tiredness, palpitations, and shortness of breath. Lab

tests show chronic hemolysis but no hematuria. A CD59 complement test is positive for a

defect to the membrane protein phosphatidylinositol glycan (PGI).

a) Gastrointestinal cancer

b) Autoimmune hemolytic anemia

Page 42: Quiz Hematology Oncology Part 1 of 2

Hematology & Oncology – Part 1 14Mar2009

DO NOT DISTRIBUTE - 42 -

c) Paroxysmal nocturnal hemoglobinuria

d) Hereditary spherocytosis

e) Thalassemia

11.1) A patient presents with cyanosis shortly after treatment for an M. pneumoniae

infection. Lab tests show anemia, spherocytes, and IgM antibody binding and clumping

to cells. Which of the following is most likely?

a) Hereditary spherocytosis

b) Cold antibody autoimmune hemolytic anemia

c) Warm antibody autoimmune hemolytic anemia

d) Cold hemolysin hemolytic anemia

e) Hemolytic-uremic syndrome

11.2) A patient being treated for syphilis presents with tiredness. Lab tests show anemia,

spherocytes, and the presence of IgG antibodies on the surface membrane of RBCs.

Which of the following is most likely?

a) Hereditary spherocytosis

b) Cold antibody autoimmune hemolytic anemia

c) Warm antibody autoimmune hemolytic anemia

d) Cold hemolysin hemolytic anemia

e) Hemolytic-uremic syndrome

12) Which of the following tests is used to diagnose autoimmune hemolytic anemia by

detecting antibodies and/or complement on RBCs, showing agglutination (clumping)?

a) Direct Coombs test (DAT)

b) Indirect Coombs test (IAT)

c) Fragment D-dimer (D-dimer)

d) Prothrombin Time (PT)

e) Activated Partial Thromboplastin Time (aPTT)

13) Which of the following tests would be used to screen for antibodies in the preparation

for a blood transfusion?

a) Direct Coombs test (DAT)

b) Indirect Coombs test (IAT)

c) Fragment D-dimer (D-dimer)

d) Prothrombin Time (PT)

e) Activated Partial Thromboplastin Time (aPTT)

14) A child presents with sudden onset of bleeding, neurological changes, and oliguria.

History reveals a recent intestinal infection with E. coli. Blood smear shows burr cells

(echinocytes). The clinician fears acute renal failure and begins treatment immediately.

What is the diagnosis?

a) Hereditary spherocytosis

b) Autoimmune hemolytic anemia

c) Paroxysmal nocturnal hemoglobinuria

d) Hemolytic-uremic syndrome

e) Thalassemia

15.1) An elderly patient presents with atrophic glottitis, a shiny, glazed, beefy tongue,

and neurological defects. Lab tests show an extreme reduction in red cell count.

Autoimmune destruction of the gastric mucosa is suspected as there is very little intrinsic

Page 43: Quiz Hematology Oncology Part 1 of 2

Hematology & Oncology – Part 1 14Mar2009

DO NOT DISTRIBUTE - 43 -

factor found. The clinician is worried about adenocarcinoma of the stomach antrum.

Which of the following is likely?

a) Vitamin K deficiency

b) Vitamin C deficiency

c) Vitamin B12 deficiency

d) Folate deficiency

e) Iron deficiency

15.2) Pernicious anemia is associated with:

a) Vitamin K deficiency

b) Vitamin C deficiency

c) Vitamin B12 deficiency

d) Folate deficiency

e) Iron deficiency

16) An alcoholic presents with diarrhea and weight loss. He states he drinks alcohol and

eats fast food burgers as his main dietary intake. Blood tests show anemia and a mean

corpuscle volume of 130fL (macrocytic/megaloblastic). Which of the following is most

likely?

a) Vitamin K deficiency

b) Vitamin C deficiency

c) Vitamin B12 deficiency

d) Folate deficiency

e) Iron deficiency

17.1) A child from a developing country presents with exhaustion. Blood tests show

hypochromic microcytic anemia with a low ferritin level and a high transferrin level. The

child complains of tongue burning, and Plummer-Vinson syndrome (PVS) is suspected.

Which of the following is most likely?

a) Vitamin K deficiency

b) Vitamin C deficiency

c) Vitamin B12 deficiency

d) Folate deficiency

e) Iron deficiency

Match the GI location with the absorbed vitamin/mineral:

17.2) Ileum a) Iron

17.3) Jejunum b) Folate

17.4) Duodenum c) Vitamin B12

18) A patient with rheumatoid arthritis presents “feeling tired.” Lab tests show a

hematocrit of 27%, hemoglobin is 1/3 of hematocrit, and mean corpuscle volume of 65fL.

Ferritin levels are high and transferrin levels are low. Serum iron is not elevated. Iron is

being stored in mononuclear phagocytic cells. Which of the following is most likely?

a) Iron deficiency anemia

b) Anemia of chronic disease

c) Sideroblastic anemia

d) Megaloblastic anemia

e) Pernicious anemia

19) Which of the following is associated with anemia, neutropenia, and

thrombocytopenia and is a reduction in all cellular elements of the blood?

Page 44: Quiz Hematology Oncology Part 1 of 2

Hematology & Oncology – Part 1 14Mar2009

DO NOT DISTRIBUTE - 44 -

a) Pure red cell aplasia

b) Myelophthisic anemia

c) Aplastic anemia

d) Polycythemia

e) Pancytopenia

20) A patient undergoing radiation therapy is put on chloramphenicol for an infection.

The patient develops progressive weakness, pallor, and dyspnea. Bone marrow aspiration

is hypocellular and lab tests show reticulocytopenia. Which of the following is likely?

a) Pure red cell aplasia

b) Myelophthisic anemia

c) Aplastic anemia

d) Polycythemia

e) Pancytopenia

21) Pure red cell aplasia is seen as marked hypoplasia of marrow erythroid elements.

What disease is this associated with?

a) Thymoma

b) Leukemia

c) Mesothelioma

d) Prostate cancer

e) Sickle cell anemia

22) A patient with prostate cancer presents with progressive weakness and dyspnea. A

bone scan shows the cancer has metastasized. Blood tests show marrow failure with

immature erythroic and myeloid progenitors appearing in the peripheral blood

(leukoerythroblastosis). Which of the following is most likely?

a) Pure red cell aplasia

b) Myelophthisic anemia

c) Aplastic anemia

d) Polycythemia

e) Pancytopenia

23) A mountain climber develops dyspnea at high altitude and is airlifted to a ground

hospital. Blood tests show an increased level of erythropoietin and reduced plasma

volume with increased hemoglobin (hemoconcentration). This patient likely has a

secondary form of which of the following?

a) Pure red cell aplasia

b) Myelophthisic anemia

c) Aplastic anemia

d) Polycythemia

e) Pancytopenia

24.1) Which of the following causes of bleeding is due to impaired formation of collagen

needed for support of vessel walls?

a) Infection and drug reactions

b) Scurvy and Ehlers-Danlos syndrome

c) Henoch-Schönlein purpura

d) Hereditary hemorrhagic telangiectasia

e) Amyloid infiltration of blood vessels

Page 45: Quiz Hematology Oncology Part 1 of 2

Hematology & Oncology – Part 1 14Mar2009

DO NOT DISTRIBUTE - 45 -

24.2) A patient presents with mucocutaneous petechiae. A lab test shows apple-green

birefringence on Congo red stain. Which of the following is most likely?

a) Infection and drug reactions

b) Scurvy and Ehlers-Danlos syndrome

c) Henoch-Schönlein purpura

d) Hereditary hemorrhagic telangiectasia

e) Amyloid infiltration of blood vessels

24.3) A patient presents with dilated, torturous blood vessels with thin walls that bleed

readily (hemorrhagic telangiectasia). What is the genetic inheritance pattern?

a) X-linked dominant

b) X-linked recessive

c) Y-linked

d) Autosomal dominant

e) Autosomal recessive

24.4) An infant presents with purpuric rash, colicky abdominal pain, and acute

glomerulonephritis. Which of the following is most likely?

a) Infection and drug reactions

b) Scurvy and Ehlers-Danlos syndrome

c) Henoch-Schönlein purpura

d) Hereditary hemorrhagic telangiectasia

e) Amyloid infiltration of blood vessels

25) Thrombocytopenia is characterized by a low platelet count (<100,000/mcL),

prolonged bleeding time, and normal PT/aPTT. What form of thrombocytopenia develops

in patients with hypersplenism?

a) Decreased platelet production

b) Decreased platelet survival

c) Sequestration

d) Dilutional

e) Neonatal

26) Which of the following is associated with previous sensitization to an antigen by

receiving platelets and, in addition to HLA and ABO antigens, may involve other

antigens such as Duzo, PL, and Bak?

a) Neonatal thrombocytopenia

b) Sequestration thrombocytopenia

c) Post-transfusional thrombocytopenia

d) Idiopathic thrombocytopenic purpura

e) Thrombotic thrombocytopenic purpura

27) A 25-year-old female presents with a sudden “shower” of petechial hemorrhages in

the skin with no apparent injury. Systemic lupus erythrematosis (SLE) is ruled out. IgG

platelet autoantibodies are found. Which of the following is most likely?

a) Neonatal thrombocytopenia

b) Sequestration thrombocytopenia

c) Post-transfusional thrombocytopenia

d) Idiopathic thrombocytopenic purpura

e) Thrombotic thrombocytopenic purpura

Page 46: Quiz Hematology Oncology Part 1 of 2

Hematology & Oncology – Part 1 14Mar2009

DO NOT DISTRIBUTE - 46 -

28) Which of the following is associated with fever, hyaline thrombi, thrombocytopenia,

microangiopathic hemolytic anemia, and transient neurologic deficits?

a) Neonatal thrombocytopenia

b) Sequestration thrombocytopenia

c) Post-transfusional thrombocytopenia

d) Idiopathic thrombocytopenic purpura

e) Thrombotic thrombocytopenic purpura

29.1) Glanzmann thrombasthenia is an autosomal recessive platelet disorder where there

is failure to respond to adenosine diphosphate (ADP), epinephrine, or thrombin. This

leads to deficient IIb/IIIa. What is the mechanism in this disorder?

a) Defect of adhesion

b) Defect of aggregation

c) Disorder of platelet secretion

d) Aspirin ingestion

e) Uremia

29.2) Bernard-Soulier syndrome is an autosomal recessive platelet disorder caused by a

deficiency in te glycoprotein complex Ib-IX. What is the mechanism in this disorder?

a) Defect of adhesion

b) Defect of aggregation

c) Disorder of platelet secretion

d) Aspirin ingestion

e) Uremia

30) A patient presents with prolonged bleeding from mucous membranes despite a

normal platelet count. A deficiency in von Willebrand factor (vWF), a carrier protein for

factor VIII, is found. If the cause is an autosomal recessive deletion or frameshift

mutation, what is the disease form?

a) Type I (1)

b) Type II (2)

c) Type III (3)

31) An African American child is sent to the ICU after massive hemorrhage following a

routine operation. The child has a history of joint hemarthroses. Hemophilia A, an X-

linked recessive disease, is diagnosed. What coagulation factor is deficient?

a) Factor II

b) Factor VII

c) Factor VIII

d) Factor IX

e) Factor X

32) A child presents with a painful swollen knee. Blood tests show prolonged aPTT and

normal PT. Hemophilia B, an X-linked recessive disease, is diagnosed. What coagulation

factor is deficient?

a) Factor II

b) Factor VII

c) Factor VIII

d) Factor IX

e) Factor X

Page 47: Quiz Hematology Oncology Part 1 of 2

Hematology & Oncology – Part 1 14Mar2009

DO NOT DISTRIBUTE - 47 -

33) Disseminated intravascular coagulation (DIC) is a serious complication either due to

release of tissue factor/thromboplastic substances (obstetric, promyelocytic leukemia,

adenocarinoma) or widespread injury to the endothelial cells (sepsis, trauma). Which of

the following is NOT increased or prolonged when doing lab screening tests for DIC?

a) Bleeding time

b) Platelet count

c) PT

d) aPTT

e) Thrombin time

34) Which of the following is associated with neutropenia, seen as a reduction in the

number of granulocytes in the peripheral blood?

a) Iron deficiency anemia

b) Anemia of chronic disease

c) Sideroblastic anemia

d) Megaloblastic anemia

e) Pernicious anemia

35.1) Which of the following is seen in allergic disorders such as asthma and hay fever?

a) Neutrophilic leukocytosis

b) Eosinophilic leukocytosis

c) Basophilic leukocytosis

d) Monocytosis

e) Lymphocytosis

35.2) Which of the following is seen in chronic infections such as tuberculosis and

endocarditis as well as collagen vascular diseases?

a) Neutrophilic leukocytosis

b) Eosinophilic leukocytosis

c) Basophilic leukocytosis

d) Monocytosis

e) Lymphocytosis

36) Which of the following is NOT associated with acute non-specific lymphadenitis?

a) Teeth infections

b) Tonsil infections

c) Meningial infections

d) Extremity infections

e) Appendix infections

37) A high-school students presents to the primary care clinic because she “feels tired.”

She has a Castell sign and dullness to percussion over the Traube space. Splenomegaly is

found. What is the most likely cause?

a) Infection (mononucleosis, endocarditis)

b) Portal hypertension (congestive)

c) Lymphohematogenous disorder

d) Immunologic-inflammatory condition

e) Storage disease (Gaucher)

38) Cirrhosis of the liver leads to what form of splenomegaly?

a) Infection (mononucleosis, endocarditis)

b) Portal hypertension (congestive)

Page 48: Quiz Hematology Oncology Part 1 of 2

Hematology & Oncology – Part 1 14Mar2009

DO NOT DISTRIBUTE - 48 -

c) Lymphohematogenous disorder

d) Immunologic-inflammatory condition

e) Storage disease (Gaucher)

39) Although all of the following can cause acute rupture of the splenic capsule, which is

the most commonly seen?

a) Mononucleosis infection

b) Malarial infection

c) Typhoid fever infection

d) Lymphoid neoplasms

e) Traumatic crush injury

40) Reticulocyte sound should be above 3% for good bone marrow response (below 2%

is poor response). Which of the following describes how to correct a reticulocyte count in

the presence of polychromasia?

a) (Hematocrit / 45) * Reticulocyte * 4

b) (Hematocrit / 45) * Reticulocyte * 2

c) Hematocrit / 45 * Reticulocyte count

d) (Hematocrit / 45) * Reticulocyte / 2

e) (Hematocrit / 45) * Reticulocyte / 4

Hematology/Oncology #20 – Nutrition

1) You are requesting a grant to screen for iron deficiency. Because of limited funds, you

want to target the population in which iron deficiency has the most devastating impact.

You choose:

a) Patients on renal dialysis

b) Alcoholics

c) Elderly adults living by themselves

d) Young children from poor communities

e) Young adults

2) How does low iron status affect pregnancy outcome?

a) It increases the risk of heart defects

b) It increases the risk of preterm birth

c) It promotes fetal macrosomia

d) It delays the onset of labor

e) It increases the risk of bleeding

3) Name a characteristic of heme iron:

a) Relatively high bioavailability

b) Found only in dietary supplements

c) Found only in red meats

d) Found only in plant foods

e) Requires reduction before absorption

4) How is non-heme iron taken up from the intestinal lumen?

a) A specific transporter takes up Fe3+ and exports it with transferring

b) Ferritin channels divalent iron into the enterocyte

c) Transferrin in the luminal membrane is an acceptor of ferric iron

d) Uptake of divalent iron proceeds via the transferring receptor

e) A transporter shared with other metals mediates Fe++ uptake

Page 49: Quiz Hematology Oncology Part 1 of 2

Hematology & Oncology – Part 1 14Mar2009

DO NOT DISTRIBUTE - 49 -

5) How is iron stored?

a) As a 2:1 complex of Fe3+ with ferritin in the bone marrow

b) As a complex of multiple Fe++ with transferring in the liver

c) As a complex with ferritin in the liver and other sites

d) As a complex of Fe++ with transferring and transferring receptor

e) As a complex with DMT1 in liver, spleen, and bone marrow

6) A 19-year-old female comes to your clinic for a pre-athletic physical. She looks

unusually pale so you do a quick dietary screening and find she rarely eats meat, but

drinks milk and tea and eats a lot of fruits and vegetables. A blood count shows that she

is anemic and that her MCV is low. Which of the following would be the BEST test to

do next?

a) Serum iron concentration

b) Serum ferritin

c) Transferrin saturation

d) Urine homocysteine

e) Schilling test

7) An elderly man comes into your clinic for a check-up. He has hemorrhoids and has

had mild anemia for the past year. He is using iron supplements. You check a blood

count, serum ferritin, and transferrin saturation, all of which are in the normal range.

Based on these findings, what is your assessment?

a) He now has a B12 deficiency that is masking the original iron deficiency

b) He now has multiple anemia and you add another supplement

c) He is still anemic and should continue the supplement

d) He is no longer anemic, but should continue high iron foods

e) He now has folate deficiency that is masking the original iron deficiency

8) A 19-year-old female comes into the office complaining of fatigue. A few tests show

that she has iron deficiency anemia. She probably has an elevated concentration of which

serum protein?

a) Transferrin

b) Myoglobin

c) Ferritin

d) Hemoglobin

e) Cytochrome

9) A 56-year-old woman comes to you for a flu shot. She lives alone and does her own

cooking. You find that she has iron deficiency anemia. What foods would you

recommend that are good sources of iron with the best absorption rate?

a) Spinach and broccoli

b) Black tea

c) Chicken, fish, and beef

d) Legumes

e) Iron fortified cereals

10) A mother brings her 5-year-old child for a school physical. The child is very pale.

You suspect iron deficiency anemia. Before doing any lab tests you decide to ask about

the intake of which foods or nutrients?

a) Bread, rice, and corn

b) Meats, legumes, cereals

Page 50: Quiz Hematology Oncology Part 1 of 2

Hematology & Oncology – Part 1 14Mar2009

DO NOT DISTRIBUTE - 50 -

c) Yellow and orange vegetables

d) Saturated fats

e) Vitamin D and calcium

11) A mother brings her 4-year-old child into your clinic for a check-up. The child is

pale and the mother notes the child seems to tire more easily than usual. You order a

blood count and find that the child has microcytic, hypochromic anemia. You prescribe

an iron containing dietary supplement for six months and encourage the mother to

increase the following foods in the child’s diet:

a) High-fiber foods

b) Low-fat dairy foods and condiments

c) Milk, yogurt, and cheeses

d) Meat, cereal, dark green vegetables, and vitamin C-rich fruits

e) Vegetables with high levels of exalates and phytates

12) Mrs. Shibata, a 48-year-old Japanese American woman, sustained the following

injuries in a car accident: broken ribs, a punctured lung, a ruptured spleen and injuries to

her small intestine. Her spleen and part of her small intestine, including the lower third,

were removed. How long will Mrs. Shibata’s vitamin B12 stores last without treatment?

a) 3 to 6 months

b) 20 to 30 months

c) 24 months

d) 3 to 6 years

e) 20 to 30 years

13) Mrs. Shibata, has her spleen and lower third of her small intestine removed following

a car accident. After you make your first estimate of Mrs. Shibata’s vitamin B12 stores, a

nutrition consult is ordered and charted. You discover that she was a vegan for 10 to 15

years before the accident. How would you revise your estimate of how long her B12

stores will last?

a) No revision needed

b) 1.5 to 3 months

c) 10 to 15 months

d) 3 to 6 years

e) 20 to 30 years

14) You have an 8-year-old patient whose laboratory values indicate folate deficiency.

What is the most likely cause?

a) A dislike of cooked vegetables

b) Increased needs from a growth spurt

c) Medications that interfere with folate metabolism

d) Celiac disease

e) Poor intake

15) A patient with leukemia comes to you for a check-up. She recently was started on

methotrexate (a dihydrofolate reductase inhibitor) to treat her leukemia. As a result,

dihydrofolate is not reduced to tetrahydrofolate. In this patient you would expect a

disruption of which metabolic process?

a) Homocysteine reduction

b) DNA synthesis

c) Folic acid transport

Page 51: Quiz Hematology Oncology Part 1 of 2

Hematology & Oncology – Part 1 14Mar2009

DO NOT DISTRIBUTE - 51 -

d) Transferrin production

e) Intrinsic factor production

16) A 25-year-old female comes to you for a PAP smear. She says she is hoping to get

pregnant and is going to discontinue using her birth control pill. You do routine lab work

and find she has an elevated MCV, a low RBC folate level, a normal serum B12, and

normal serum ferritin. Which nutritionally related health risk to the fetus should you be

most concerned bout at this point?

a) Neural tube defects

b) Stunted growth

c) Pernicious anemia

d) Malabsorption

e) Decreased learning ability

17) As a coenzyme, a folate metabolite usually has what form?

a) Monoglutamyl dihydrofolic acid

b) Monogluyamyl folic acid

c) Polyglutamyl tetrahydrofolic acid

d) Polyglutamyl folic acid

e) Polyglutamyl dihydrofolic acid

18) Which is usually the earliest sign of folate deficiency?

a) Low RBC folate concentration

b) Low hemoglobin concentration

c) Hypersegmented neutrophils

d) Low serum folate concentration

e) Increased MCV

19) A 47-year-old woman is found to have macrocytosis. Upon further investigation, a

folate deficiency is diagnosed. The macrocytosis is the result of impaired synthesis of

which compound?

a) DNA

b) Dihydrofolate

c) Hemoglobin

d) Methylmalonic acid

e) Homocysteine

20) A patient comes to you complaining of diarrhea. After much lab work, you find that

she has macrocytic anemia, excessive blood homocysteine concentration, normal urinary

excretion of methylmalonic acid, and hypersegmented white blood cells. What is your

diagnosis?

a) Hypochromic anemia

b) Iron deficiency anemia

c) Pernicious anemia

d) Vitamin B12 deficiency

e) Folate deficiency

21) A 44-year-old woman comes to you for a check-up. You find that she is anemic and

has an elevated MCV with a normal serum B12 but a low RBC folate. What is the BEST

next step?

a) Suggest high dose of folate supplements for one year

b) Suggest the daily intake of an RDA-level multivitamin supplement

Page 52: Quiz Hematology Oncology Part 1 of 2

Hematology & Oncology – Part 1 14Mar2009

DO NOT DISTRIBUTE - 52 -

c) Encourage the intake of well-cooked meats

d) Give monthly folate injections

e) Give 2 mg folic acid supplements daily for one month

22) You suspect that a patient has folate deficiency. You would want to ask about his

intake of which foods?

a) Fats and oils

b) Seafood and poultry

c) Green leafy vegetables, fortified cereals, and citrus fruits

d) Eggs, milk, and dairy products

e) High-fiber foods like bran, carrots, and apples

23) A 73-year-old man comes to you with complains of being “worn out”. You find that

he has elevated MCV and low hemoglobin. You ask about his diet and find that he eats a

balanced diet that includes meat, milk, green leafy vegetables, and citrus fruits. What

type of nutritional deficiency is he most likely to have and why?

a) Vitamin B12 deficiency due to changes with aging

b) Vitamin B12 deficiency due to inadequate intake

c) Folate deficiency due to inadequate intake

d) Iron deficiency due to absorptive changes with aging

e) Iron deficiency due to inadequate intake

24) You are going to be doing nutritional screening at a health fair to be held in the local

mall. Which of the following individuals would be least likely to have a vitamin B12

deficiency?

a) A 55-year-old males with chronic gastritis

b) A 22-year-old male with no health complaints

c) A 43-year-old female who has been a vegan since age 16

d) A 72-year-old male with no health complaints

e) An 88-year-old female with mild heart disease

25) A 21-year-old pregnant female comes for her first prenatal appointment. When you

ask about her diet you find she has eaten no animal products for the past 6 years. She did

take a folate supplement prior to becoming pregnant and during her first trimester. She is

now taking an iron supplement. Lab work shows she is not anemic. Nonetheless, her

infant could be at risk for:

a) Vitamin B12 deficiency

b) Folate deficiency

c) Stunted fetal growth

d) Iron deficiency

e) Reduced secretion of intrinsic factor

26) A 27-year-old woman comes to you for a check-up. She has intestinal inflammatory

disease (Crohn’s disease), which causes chronic GI bleeding. She has recently had most

of her ileum removed. Which nutritional deficiency or combination of deficiencies is

most likely to occur?

a) Iron and vitamin B12 deficiency

b) Folate deficiency

c) Vitamin B12 deficiency

d) Folate and B12 deficiency

e) Folate and iron deficiency

Page 53: Quiz Hematology Oncology Part 1 of 2

Hematology & Oncology – Part 1 14Mar2009

DO NOT DISTRIBUTE - 53 -

27) A 61-year-old man has just had a surgical resection of his entire stomach because of

cancer. This type of surgery puts him at risk for a nutritional anemia because of lack of:

a) Lactoferrin

b) Transobalamin

c) Ghrelin

d) Intrinsic factor

e) R factors

28) Antibodies against intrinsic factor can cause which of the following nutritional

deficiencies?

a) Combined folate and vitamin B12

b) Folate

c) Iron

d) Vitamin B12

e) Combined iron and folate

29) A 32-year-old woman had a resection of her terminal ileum (due to severe intestinal

inflammation from Crohn’s disease), which results in bile acid malabsorption. What

nutrient should you be concerned about and how quickly will a deficiency develop?

a) Folate deficiency in weeks

b) Vitamin B12 deficiency in a few months

c) Iron deficiency in one month

d) Vitamin B12 deficiency in decades

e) Folate deficiency in years

30) A 72-year-old house bound woman has a vitamin B12 deficiency due to greatly

decreased intrinsic factor. Which long-term regimen will probably be sufficient and is

most cost effective?

a) Intrinsic factor supplement daily

b) Daily oral folate

c) Daily consumptions of 2-3 servings of meat, milk, and /or eggs

d) High dose oral B12 supplement daily

e) Daily injections of a combination of vitamin B12 and tetrahydrofolate

31) A mother comes to you because she is worried about her daughter who does not want

to eat meat. She wants to know if the 15-year-old will become vitamin B12 deficient.

You tell her that vitamin B12 deficiency will NOT occur if the daughter includes the

following foods in her diet on a daily basis:

a) Lots of vegetables and nuts

b) Whole grains and legumes

c) Milk, cheese, or eggs

d) A balance of beans and vegetables

e) Soy milk and tofu along with grains, vegetables, and fruits

Hematology/Oncology #21 – Clinical: Anemia

1) What is the lower limit of normal venous hemoglobin concentration as defined by the

World Health Organization (WHO) for males and females?

a) 12g/dL and 11g/dL

b) 11g/dL and 12g/dL

c) 13g/dL and 12g/dL

Page 54: Quiz Hematology Oncology Part 1 of 2

Hematology & Oncology – Part 1 14Mar2009

DO NOT DISTRIBUTE - 54 -

d) 12g/dL and 13g/dL

e) 14g/dL and 13g/dL

2) What is the test performed in the initial evaluation of anemia after the patient history

and physical exam?

a) Bleeding time

b) Platelet count

c) Complete blood count (CBC)

d) Prothrombin time (PT)

e) Partial thromboplastin time (PTT)

3) What is the most common type of anemia?

a) Macrocytic anemia

b) Normocytic anemia

c) Microcytic anemia

d) Dimorphic anemia

e) Heinz body anemia

4) Which of the following would NOT be included on the differential diagnosis for

hypochromic microcytic anemia?

a) Sideroblastic anemia

b) Anemia of chronic disease

c) Iron deficiency anemia

d) Thalassemia

e) Pernicious anemia

5) Which of the following causes of iron deficiency anemia (sideropenic anemia) is the

most commonly seen in premenopausal women?

a) Blood loss (trauma, ulcer)

b) Hookworm infection

c) Pregnancy

d) Menstrual bleeding

e) Malabsorption or poor diet

6) What type of anemia seen in non-referral practice is the cause of up to 90% of all

hypochromic microcytic anemias?

a) Iron deficiency anemia

b) Anemia of chronic disease

c) Sideroblastic anemia

d) Lead poisoning

e) Thalassemia

7) What is the most useful initial test in documenting iron deficiency?

a) Complete blood count

b) Serum ferritin

c) Serum transferrin

d) Serum iron

e) Total iron binding capacity (TIBC)

8) Serum ferritin would NOT be increased in which of the following, in the presence of

iron deficiency?

a) Rheumatoid arthritis

b) Liver disease

Page 55: Quiz Hematology Oncology Part 1 of 2

Hematology & Oncology – Part 1 14Mar2009

DO NOT DISTRIBUTE - 55 -

c) Ankylosing spondylitis

d) Hepatocellular carcinoma

e) Malignancy

9) Patients with iron deficiency anemia should take ferrous sulfate tables (325mg) how

often?

a) Weekly

b) Daily

c) Twice a day

d) Three times a day

e) Four times a day

10) In order to replenish bone marrow reserves, what is the appropriate length of

treatment for iron deficiency anemia?

a) 1 week

b) 6 weeks

c) 2.5 months

d) 7.5 months

e) 1.5 years

f) For life

11) Along with patients on renal dialysis, which of the following patients would be

indicated for the use of intravenous iron therapy?

a) Patient undergoing cardiothoracic bypass surgery

b) Pregnant patient with iron deficiency anemia

c) Chemotherapy patient with low iron

d) Patient who received atomic radiation due to a bomb

e) Jehovah’s Witness follower undergoing surgery

12.1) Lab values return for a patient with possible microcytic anemia. The results are

decreased mean corpuscular volume (MCV), increased RBCs, TIBC 250mcg/dL, normal

ferritin, and normal marrow iron. Which of the following is most likely?

a) Iron deficiency anemia

b) Anemia of chronic disease

c) Sideroblastic anemia

d) Lead poisoning

e) Thalassemia

12.2) Lab values return for a patient with possible microcytic anemia. The results are

normal mean corpuscular volume (MCV), decreased RBCs, TIBC 250mcg/dL, increased

ferritin with 17% transferritin saturation, and increased marrow iron. Which of the

following is most likely?

a) Iron deficiency anemia

b) Anemia of chronic disease

c) Sideroblastic anemia

d) Lead poisoning

e) Thalassemia

12.3) Lab values return for a patient with possible microcytic anemia. The results are

decreased mean corpuscular volume (MCV), decreased RBCs, TIBC 350mcg/dL, low

ferritin with 7% transferritin saturation, and absent marrow iron. Which of the following

is most likely?

Page 56: Quiz Hematology Oncology Part 1 of 2

Hematology & Oncology – Part 1 14Mar2009

DO NOT DISTRIBUTE - 56 -

a) Iron deficiency anemia

b) Anemia of chronic disease

c) Sideroblastic anemia

d) Lead poisoning

e) Thalassemia

13) Which of the following would result in an increase in alpha-globulin chains in RBCs?

a) Iron deficiency anemia

b) Anemia of chronic disease

c) Sideroblastic anemia

d) Alpha-Thalassemia (minor)

e) Beta-Thalassemia

14.1) Which of the following would be seen in a patient with beta-thalassemia?

a) Low or normal MCV with a normal hemoglobin concentration

b) High MCV with low hemoglobin concentration

c) Mild anemia with marked microcytosis

d) Anemia with marked macrocytosis

e) Drastically increased transferritin saturation

14.2) Which of the following would be seen in a patient with alpha-thalassemia?

a) Low or normal MCV with a normal hemoglobin concentration

b) High MCV with low hemoglobin concentration

c) Mild anemia with marked microcytosis

d) Anemia with marked macrocytosis

e) Drastically increased transferritin saturation

15) A homeless man presents with large purpura, especially around the hair follicles and

on the legs. Examination reveals mucous membrane bleeding and spongy gums. Some

teeth are missing and the patient reports that they recently started falling out. Purple dots

are found on the tongue. The patient reports diarrhea. Blood tests reveal anemia. What

vitamin deficiency is most likely?

a) Vitamin A

b) Folate

c) Vitamin B12

d) Vitamin C

e) Vitamin E

16) Which of the following would NOT be included on the differential diagnosis for

macrocytic anemia?

a) Vitamin B12 deficiency

b) Folate deficiency

c) Hyperthyroidism

d) Alcohol abuse

e) Liver disease

17) Which of the following would NOT cause macrocytosis?

a) Alcohol

b) Antiarrythmics

c) Chemotherapy agents

d) Oral contraceptives

e) Anticonvulants

Page 57: Quiz Hematology Oncology Part 1 of 2

Hematology & Oncology – Part 1 14Mar2009

DO NOT DISTRIBUTE - 57 -

18) What vitamin deficiency is associated with pernicious anemia, bacterial overgrowth

syndromes, gastrectomy, ileal resection, achlorohydria, and chronic pancreatitis?

a) Vitamin A

b) Folate

c) Vitamin B12

d) Vitamin C

e) Vitamin E

19) An alcoholic patient presents with diarrhea and gait disturbance. Ankle reflexes are

absent. Position sense is impaired. Physical exam reveals a beefy, atrophic tongue. A

blood smear is shown here. Which of the following is most likely?

a) Vitamin A deficiency

b) Folate deficiency

c) Vitamin B12 deficiency

d) Vitamin C deficiency

e) Vitamin E deficiency

20) Which of the following laboratory findings would be diagnostic for a vitamin B12

deficiency?

a) B12 serum level of 200pg/mL

b) B12 serum level of 100pg/mL

c) Increased urinary methylmalonic acid

d) Abnormal intrinsic factor antibody

e) Positive blocking antibody test

21.1) The Schilling test, not often used today, is a two-part test designed to determine the

cause of a vitamin B12 deficiency. Part one involves radiolabled B12 ingestion and B12

IM injection with urine measurement. Part two involves intrinsic factor. Which of the

following Schilling test results is consistent with pernicious anemia?

a) Normal part one & Normal part two

b) Normal part one & Low part two

c) Low part one & Normal part two

d) Low part one & Low part two

e) High part one & Normal part two

21.2) Which of the following Schilling test results is consistent with B12 malabsorption?

a) Normal part one & Normal part two

b) Normal part one & Low part two

c) Low part one & Normal part two

d) Low part one & Low part two

e) High part one & Normal part two

22) For patients with pernicious anemia, vitamin B12 supplementation is required either

in high-dose oral tablet form or in IM injection form (500 to 1,000mcg). How long

should treatment last?

a) 1 week

b) 6 weeks

c) 2.5 months

d) 7.5 months

e) 1.5 years

f) For life

Page 58: Quiz Hematology Oncology Part 1 of 2

Hematology & Oncology – Part 1 14Mar2009

DO NOT DISTRIBUTE - 58 -

23) Megaloblastic anemia due to folate deficiency can occur in pregnancy, alcoholism, or

disease to what part of the GI tract?

a) Esophagus

b) Stomach

c) Duodenum

d) Small intestine

e) Large intestine

24) Stem cell dysfunction, kidney disease, hemolysis, mixed nutritional deficiency,

chemotherapy, rheumatoid arthritis, and acute disease anemia all fall under what broad

category of anemias?

a) Macrocytic anemia

b) Normocytic anemia

c) Microcytic anemia

d) Dimorphic anemia

e) Aplastic anemia

25) What test is performed to exclude hemolysis in the evaluation of normochromic

normocytic anemia?

a) Reticulocyte count

b) Serum ferritin

c) Serum transferrin

d) Complete blood count (CBC)

e) Soluble transferrin receptor concentration (sTfR)

26) What lab test differentiates iron deficiency anemia from anemia of chronic disease?

a) Reticulocyte count

b) Serum ferritin

c) Serum transferrin

d) Complete blood count (CBC)

e) Soluble transferrin receptor concentration (sTfR)

27) What organ is responsible for producing 90% of the body’s erythropoietin?

a) Liver

b) Spleen

c) Thymus

d) Kidney

e) Adrenals

28) Which of the following would NOT have low erythropoietin levels?

a) HIV infection

b) Rheumatoid arthritis (RA)

c) Systemic lupus erythematosus (SLE)

d) Polycythemia rubra vera (PRV)

e) Chronic renal failure

29) Which of the following would NOT have high erythropoietin levels?

a) Zidovudine therapy

b) Vitamin B12 therapy

c) Marrow hypofunction

d) Malignancy

e) Hypoxia

Page 59: Quiz Hematology Oncology Part 1 of 2

Hematology & Oncology – Part 1 14Mar2009

DO NOT DISTRIBUTE - 59 -

30) Erythropoietin therapy is indicated in all of the following patients EXCEPT:

a) Anemia due to malignancy

b) Anemia caused by renal insufficiency

c) Anemia due to HIV infection

d) Anemia due to zidovudine therapy

e) Anemia due to folate deficiency

31) Erythropoietin therapy would be ineffective in all of the following cases EXCEPT:

a) Aluminum toxicity

b) Hyperparathyroidism

c) Anemia due to vitamin B12 deficiency

d) Anemia due to neoplastic agents

e) Iron deficiency anemia

32) Which of the following is most likely to occur with misuse of erythropoietin?

a) Hypertension

b) Diabetes

c) Thrombocytopenia

d) Meningitis

e) Sydenham chorea

33) Which of the following is characterized by peripheral pancytopenia, bone marrow

hypocellularity, absence of malignant or myeloproliferative disease at the time of

diagnosis, and is only cured by allogeneic stem cell transplantation?

a) Macrocytic anemia

b) Normocytic anemia

c) Microcytic anemia

d) Dimorphic anemia

e) Aplastic anemia

34) What is the major criteria for diagnosing aplastic anemia?

a) Neutrophil count less than 0.5x10^9/L

b) Platelet count less than 20x10^9/L

c) Bone marrow cellularity less than 25%

d) Corrected reticulocyte count less than 1%

e) Normal cytogenetic findings

35) Which of the following symptoms seen in aplastic anemia is due to leukocytopenia?

a) Weakness

b) Infections

c) Pallor and fatigue

d) Increased bruising

e) Increased bleeding

36) What is the most common cause of aplastic anemia?

a) Drugs containing gold

b) Chloramphenicol

c) Methylphenylethylhydantoin

d) Benzene

e) Idiopathic

37) Which of the following is NOT true for transfusion therapy in patients with aplastic

anemia?

Page 60: Quiz Hematology Oncology Part 1 of 2

Hematology & Oncology – Part 1 14Mar2009

DO NOT DISTRIBUTE - 60 -

a) Every patient should undergo HLA typing

b) Selecting a family member for transfusion would decrease rejection risk

c) If transfusion is necessary, select a nonrelated HLA-matched donor and use

CMV-negative, leukocyte-poor RBC transfusions and single-donor platelet

transfusions to maintain the platelet count greater than 10x10^9/L

d) Family members increase the likelihood of sensitizing the donor to minor

histocompatibility agents

38) What is the treatment of choice for patients with aplastic anemia who are older than

40 years of age?

a) Chloramphenicol

b) Blood transfusion

c) Bone marrow transplant

d) Spleen removal

e) Immunosupression

39) An allogeneic bone marrow transplant for patients with aplastic anemia should be

considered immediately in which of the following cases?

a) Patient is younger than 20 years of age and has a homozygous twin

b) Patient is younger than 20 years of age and has a heterozygous twin

c) Patient is older than 40 years of age and has a homozygous twin

d) Patient is older than 40 years of age and has a heterozygous twin

e) Patient is older than 60 years of age and is at high risk

40) Which of the following would NOT be increased in a patient with hemolytic anemia?

a) Reticulocyte count

b) Indirect bilirubin

c) Platelet count

d) Lactate dehydrogenase (LDH)

41) Along with a complete blood count, which of the following is a key component to the

initial evaluation of suspected hemolytic anemia?

a) Reticulocyte count

b) Serum ferritin

c) Serum transferrin

d) Lactate dehydrogenase (LDH)

e) Soluble transferrin receptor concentration (sTfR)

42) Which of the following is associated with alcohol burns, Clostridium infections,

autoimmune hemolytic anemia, hypophosphatemia, and hereditary spherocytosis?

a) Spur cells (acanthocytes)

b) Target cells

c) Hypochromia

d) Spherocytes

e) Basophilic stippling

43) Which of the following is associated with lead poisoning, arsenic poisoning, and

beta-thalassemia?

a) Spur cells (acanthocytes)

b) Target cells

c) Hypochromia

d) Spherocytes

Page 61: Quiz Hematology Oncology Part 1 of 2

Hematology & Oncology – Part 1 14Mar2009

DO NOT DISTRIBUTE - 61 -

e) Basophilic stippling

44) Which of the following is associated with lead poisoning and beta-thalassemia (but

not arsenic poisoning)?

a) Spur cells (acanthocytes)

b) Target cells

c) Hypochromia

d) Spherocytes

e) Basophilic stippling

45) Which of the following is associated with thalassemia, hemoglobin C and E,

obstructive jaundice, hepatitis, lecithin-cholesterol acyltransferase deficiency, and the

splenectomy state?

a) Spur cells (acanthocytes)

b) Burr cells (echinocytes)

c) Heinz bodies

d) Howell-Jolly bodies

e) Target cells

46) Which of the following is associated with chronic liver disease, abetalipoproteinemia,

malabsorption, and anorexia nervosa?

a) Spur cells (acanthocytes)

b) Burr cells (echinocytes)

c) Heinz bodies

d) Howell-Jolly bodies

e) Target cells

47) Burr cells (echinocytes) are exclusive to what disease?

a) Glucose-6-phosphase dehydrogenase deficiency

b) Alpha-thalassemia

c) Folate deficiency

d) Uremia (TTP/HUS)

e) Anorexia nervosa

48) Bite cells and Heinz bodies are seen in what disease?

a) Glucose-6-phosphase dehydrogenase deficiency

b) Alpha-thalassemia

c) Folate deficiency

d) Uremia (TTP/HUS)

e) Anorexia nervosa

49) Which of the following is associated with hyposplenism, asplenia, and possibly

megaloblastic anemia?

a) Spur cells (acanthocytes)

b) Burr cells (echinocytes)

c) Heinz bodies

d) Howell-Jolly bodies

e) Target cells

50) A positive Coombs test indicates the presence of C3 and/or what immunoglobulin on

the surface of a RBC?

a) IgA

b) IgD

Page 62: Quiz Hematology Oncology Part 1 of 2

Hematology & Oncology – Part 1 14Mar2009

DO NOT DISTRIBUTE - 62 -

c) IgE

d) IgG

e) IgM

51) Although the cause of a positive Coombs test is unknown in 95% of hospitalized

patients, it is most likely due to:

a) Anemia due to drugs

b) Ovarian cancer

c) Hemolytic anemia

d) Non-Hodgkin lymphoma

e) Chronic lymphocytic leukemia

52) For patients with warm autoimmune hemolytic anemia, transfusion is normally

avoided. However, packed RBCs may be transfused in which of the following cases?

a) The anemia is due to drugs

b) The patient is symptomatic and oxygen saturation is > 90%

c) The patient is symptomatic and oxygen saturation is < 90%

d) The patient is symptomatic and hemoglobin is > 8g/dL

e) The patient is symptomatic and hemoglobin is < 8g/dL

53) What is the treatment of choice for patients with idiopathic autoimmune hemolytic

anemia?

a) Chloramphenicol

b) Blood transfusion

c) Bone marrow transplant

d) Spleen removal

e) Corticosteroids

54) A patient with chronic lymphoid leukemia presents in January with pain in their

hands. Their fingers and toes are dusky blue and blanches. All digits are affected equally

and Raynaud syndrome is ruled out. Agglutination of RBCs is seen at cold temperatures.

Which of the following treatments would be the least effective?

a) Danazol (Danocrine)

b) Cyclophosphamide (Cytoxan)

c) Prednisone (Deltasone)

d) Interferon alfa-2b (Intron-A)

e) Vitamin B12

55.1) An child of an African American father and Mediterranean mother presents with

possible anemia. History reveals the child eats a great deal of fava beans. A glucose-6-

phosphase dehydrogenase (G6PD) deficiency is suspected. What is the genetic

inheritance pattern?

a) X-linked

b) Y-linked

c) Autosomal dominate

d) Autosomal recessive

e) Autosomal dominant with incomplete penetrance

55.2) Which of the following drugs is NOT associated with a G6PD deficiency?

a) Dapsone

b) Chloroquine

c) Aspirin

Page 63: Quiz Hematology Oncology Part 1 of 2

Hematology & Oncology – Part 1 14Mar2009

DO NOT DISTRIBUTE - 63 -

d) Tylenol

e) Sulfonamides

56) Which of the following is the definitive test for diagnosing a G6PD deficiency?

a) G6PD assay

b) Presence of bite cells

c) Presence of Heinz bodies

d) Positive intravascular hemolysis

e) Methemoglobinemia (metHb) and methemalbuminemia

57) A patient presents with jaundice. Physical exam reveals splenomegaly. Lab results

show a negative Coombs test, spherocytosis, and increased osmotic fragility. Further

exam reveals gallstones. Genetic spherocytosis is diagnosed. What is the inheritance

pattern?

a) X-linked

b) Y-linked

c) Autosomal dominate

d) Autosomal recessive

e) Autosomal dominant with incomplete penetrance

58) A patient presents with chronic hemolytic anemia and hemoglobinuria. At this visit,

splenomegaly is found as well as relative reticulocytosis. The patient complains of severe

abdominal pain, fever, vomiting, and tenderness in the right upper quadrant on palpation.

Paroxysmal nocturnal hemoglobinuria is diagnosed. What will be the likely cause of

death for this patient?

a) Irreversible hypotension due to fluid loss

b) Venous thrombosis in the portal system or brain

c) Rupture of the liver or a hepatic artery

d) Hypoxia due to decreased oxygen carrying capacity

e) The patient will die of natural causes

59) What is the initial treatment for paroxysmal nocturnal hemoglobinuria?

a) Heparin

b) Warfarin

c) Danazol

d) Dapsone

e) Prednisone

60) Which of the following is NOT a major symptom seen in thrombotic

thrombocytopenia purpura (TTP, Moschcowitz disease)?

a) Anemia

b) Thrombocytopenia

c) Neurologic signs

d) Hypertension

e) Kidney abnormalities

61) What is the primary diagnostic criteria for TPP?

a) Anemia

b) Thrombocytopenia

c) Fever

d) Platelet count

e) Bleeding time

Page 64: Quiz Hematology Oncology Part 1 of 2

Hematology & Oncology – Part 1 14Mar2009

DO NOT DISTRIBUTE - 64 -

62) Recent studies have shown a possible link between TTP and the enzyme

ADAMTS13 related to von Willebrand factor. However, the most common etiology for

TTP is:

a) Pregnancy

b) Oral contraceptives

c) Bone marrow transplant

d) Platelet aggregation inhibitors

e) Idiopathic

63) Which of the following is specifically NOT seen in TTP?

a) Microvascular inflammation

b) Platelet aggregates

c) Intraluminal fibrin

d) Hyaline vascular occlusions

64) With treatment of TTP, about 70-80% of patient will survive with little or no

sequelae. About what percentage will die of multi-organ failure without treatment?

a) 25%

b) 45%

c) 60%

d) 75%

e) 90%

65) A patient presents with microangiopathic hemolytic anemia and renal

microangiopathy. Creatine is measured above 3mg/dL. No fever or neurological signs are

seen. Adult hemolytic uremia syndrome (HUS) is diagnosed. What is the likely cause?

a) Idiopathic

b) Autoimmune reaction

c) Salmonella or E. coli

d) E. coli or Shigella

e) S. aureus or Salmonella

66) Which of the following would NOT promote sickling of RBCs?

a) Low oxygen tension

b) High pH

c) High cellular hemoglobin concentration

d) Loss of water

e) Hemoglobinopathy

67) Bone scans of patients after sickle cell anemia crisis can show bone infarcts and

periostitis. These bone crises usually affect what areas?

a) Fingers and toes

b) Wrists and ankles

c) Elbows and knees

d) Shoulders and hips

e) Middle and lower back

68) What is the origin of abdominal crisis with sickle cell anemia?

a) Spleen infarct

b) Liver infarct

c) Mesentery infarct

d) Duodenal infarct

Page 65: Quiz Hematology Oncology Part 1 of 2

Hematology & Oncology – Part 1 14Mar2009

DO NOT DISTRIBUTE - 65 -

e) Unknown (idiopathic)

69) Which of the following is used in the management of hemorrhagic strokes and TIAs

in sickle cell anemia?

a) Angiography

b) Heparin

c) Warfarin

d) Aspirin

e) RBC transfusion

70) What is the primary cause of acute chest syndrome and pulmonary crisis in adults

with sickle cell anemia as well as the secondary cause in children?

a) Salmonella

b) Haemophilus

c) Pneumococcus

d) Mycoplasma

e) Cell sickling

71) An adult presents with aplastic crisis following a febrile illness. Lab test show

reticulocytes and normoblasts are gone (reticulocytopenia). Which of the following

related infections is more common in adults than children, most likely leading to this

illness exacerbation?

a) Salmonella

b) Streptococcus

c) Epstein-Barr virus

d) B19 parvovirus

72) Infection crisis with sickle cell anemia is the most frequent cause of death in patient

younger than 5 years of age. Which of the following is NOT a common cause of

osteomyelitis associated with sickle cell anemia?

a) Acid fast bacteria

b) Staphylococcus

c) Salmonella

d) Pneumococcus

e) Gram-negative rods

73) Which of the following is NOT characteristic of lab findings in sickle cell anemia?

a) Morphology of sickled cells or cigar cells

b) Anemia between 5.5 and 9.5g/dL

c) Hyposplenia with Howell-Jolly bodies

d) Decreased white blood cell count

e) Increased liver function tests

74) Which of the following would be a poor choice for a patient with sickle cell anemia?

a) Acetaminophen for high fever

b) Influenza A and Haemophilus vaccination

c) Pneumococcal vaccination

d) Iron chelation if transfusion requirement is high

e) Aspirin for mild fever

75) With sickle cell disease, which of the following is the major indication for short-term

transfusion therapy?

a) Priapism

Page 66: Quiz Hematology Oncology Part 1 of 2

Hematology & Oncology – Part 1 14Mar2009

DO NOT DISTRIBUTE - 66 -

b) Acute chest syndrome

c) Protracted hematuria

d) Chronic skin ulcers

e) Pregnancy

76) For sickle cell patients with vaso-occlusive crisis, fluid therapy is given aimed at

correcting the urinary loss of what ion?

a) K+

b) Cl-

c) Na+

d) Ca++

e) NH3+

77) What medication has been shown to decrease the frequency of painful vaso-occlusive

crises in sickle cell disease?

a) Hydroxyurea (Hydrea)

b) Actinomycin (Actinomycin-D)

c) Doxorubicin (Adriamycin)

d) Daunorubicin (DaunoXome)

e) Bleomycin (Blenoxane)

78) A patient with sickle cell anemia has a history of cerebrovasular accidents, recurrent

chest syndromes, and progressive retinopathy. Exchange transfusion as a curative is

refused. What other option does this patient have for a cure to their sickle cell disease?

a) Immunosupression

b) Blood transfusion

c) Bone marrow transplant

d) Spleen removal

e) There is no other cure

79) Which of the following is NOT associated with increased risk of death in sickle cell

patients older then 20 years of age?

a) Acute chest syndrome

b) Renal failure

c) Seizures

d) Low WBC count

e) Low level of fetal hemoglobin

80) What is the most common cause of death in sickle cell disease?

a) Stroke

b) Septicemia

c) Uncontrollable bleeding

d) Myocardial infarction

e) Acute pain and chest syndrome

81) Which of the following is NOT seen in people with the sickle cell trait (8% of

African American population)?

a) Splenic infarction at high altitude

b) Hematuria and pulmonary embolism

c) Pyelonephritis with pregnancy

d) Glaucoma

e) Increased mortality with P. falciparum infection

Page 67: Quiz Hematology Oncology Part 1 of 2

Hematology & Oncology – Part 1 14Mar2009

DO NOT DISTRIBUTE - 67 -

82) Drug-induced neutropenia is a risk most correlated to macrolide antibiotics and:

a) NSAIDs

b) Anticonvulsants

c) Antithyroid medications

d) Cardiovascular drugs

e) Antipsychotics

83) What is the most common cause of acute hemolytic transfusion reactions?

a) Idiopathic

b) Medical error

c) Minor histocompatibility complex interactions

d) Spontaneous HLA mutations

e) Infection

84) Which of the following patients is at the highest risk for an acute hemolytic

transfusion reaction?

a) Middle-aged woman who has had children and is receiving 3 units of blood

b) Middle-aged woman who has not had children and is receiving 1 unit of blood

c) Elderly man who is receiving 1 unit of blood

d) Young boy with a rapid heart rate who is receiving 3 units of blood

e) Young girl who regularly donates blood

85) A patient receiving a blood transfusion develops pain at the IV site, apprehension,

flushing, dyspnea, and abdominal pain. Monitoring the patient shows decreased urine

output (oligouria). The transfusion is immediately terminated and vigorous administration

of fluid begins. What drug can be given to increase cortical blood flow in this patient?

a) Aspirin

b) Furosemide

c) Lithium salt

d) Thiazide

e) Mannitol

86) An adult patient begins to show signs of respiratory distress about 3 hours after a

blood transfusion. He has fever and hypotension on exam. Chest x-ray reveals pulmonary

infiltrates. What is the recommended treatment method for this patient?

a) Immediate ventilator support and high dose dopamine

b) Fluid administration and dobutamine

c) Supportive care and oxygen

d) Mannitol and monitoring

e) Blood chelation

87) Several hours after the beginning of a blood transfusion, a patient has tightness in

their chest, a dry cough, and edema. Fluid overload is suspected. Which of the following

is NOT a treatment option for this patient?

a) Slowing the transfusion to 100mL an hour

b) Placing the patient in a sitting position

c) Administering diuretics

d) Re-typing the patient’s ABO blood

88) A patient presents with a yellow-brown skin pigment. Blood tests shown anemia,

thrombocytopenia, and abnormal cells. The abnormal cells are large, have an eccentric

nucleus, and fibrillary cytoplasm giving a tissue paper appearance (shown). X-ray reveals

Page 68: Quiz Hematology Oncology Part 1 of 2

Hematology & Oncology – Part 1 14Mar2009

DO NOT DISTRIBUTE - 68 -

lesions of the femur giving it an Erlenmeyer flask appearance. Painless heptatomegaly

and splenomegaly are found. A deficiency of the enzyme glucocerebrosidase is found.

Gaucher disease is diagnosed. Which of the following treatment options has been largely

replaced today?

a) Allogeneic bone marrow transplant

b) Hemisplenectomy

c) Miglustat (Zavesca)

d) Imiglucerase (Cerezyme)

e) Alglucerase (Ceredase)

89.1) A patient presents with blistering skin lesions covering light-exposed areas. The

skin areas show hyperpigmentation and hypertricosis. Uroporphyrins are increased in the

urine. Blood test show a heptatitis C infection of new onset. The patient reports heavy

drinking of alcohol. Phlebotomy is prescribed to remove iron as well as chloroquine.

Which of the following is likely?

a) Porphyria cutanea tarda (PCT)

b) Acute intermittent porphyria

c) Erythropoietic porphyria

d) Guillain-Barré syndrome

e) Drug photosensitivity

89.2) A patient presents with 5 days of abdominal pain, polyneuropathy, motor paralysis,

hallucinations, and seizures. Lab studies show normal protoporphyrin and increased

urinary delta-aminolevulinic acid (ALA). The patient had recently begun a course of

antibiotic therapy with sulfonamides but did not stop drinking alcohol as requested. The

patient is told to eat large amounts of carbohydrates, avoid fasting, and is educated about

intravenous hematin treatment. Which of the following is likely?

a) Porphyria cutanea tarda (PCT)

b) Acute intermittent porphyria

c) Erythropoietic porphyria

d) Guillain-Barré syndrome

e) Drug photosensitivity

90) What is the hallmark lab finding in AIDS?

a) Pancytopenia

b) Neutropenia

c) Thrombocytopenia

d) Lymphocytopenia

e) Aplastic anemia

91) What is the most common cause of neutropenia in patients with AIDS?

a) TMP-SMX

b) Antineoplastic chemotherapy

c) Gancyclovir or pentamidine

d) Decreased production of neutrophils

e) Autoimmune destruction of antigranulocyte antibodies

92) Which of the following is true in AIDS patients?

a) Most have increased erythropoietin levels

b) The type of anemia is normochromic normocytic anemia

c) Transfusion will likely increase survival

Page 69: Quiz Hematology Oncology Part 1 of 2

Hematology & Oncology – Part 1 14Mar2009

DO NOT DISTRIBUTE - 69 -

d) Transfusion decreases the risk of CMV infection

93) Kaposi sarcoma (HHV8) is the most commly seen HIV associated malignancy. What

is the second most common?

a) Leiomyosarcoma

b) Bronchogenic carcinoma

c) Non-melanoma skin cancer

d) Non-Hodgkin’s lymphoma

e) Plasma cell neoplasms

Hematology/Oncology #22 – Clinical: Coagulation

1) What lab study is done initially in the evaluation of bleeding disorders?

a) Activated partial thromboplastin time (aPTT)

b) Prothrombin time (PT)

c) vWD and XIII assay

d) Fibrinogen

e) CBC

2) Which of the following is part of the intrinsic clotting pathway, and thus would NOT

be evaluated with a prothrombin time (PT) test?

a) Factor VII

b) Factor V

c) Factor X

d) Factor XI

e) Fibrinogen

3) Which of the following is part of the intrinsic clotting pathway, and thus would NOT

be evaluated with an activated partial thromboplastin time (aPTT) test?

a) Factor XII

b) Factor IX

c) Factor VII

d) Factor XI

e) Factor VIII

f) Factor V

4) Factor deficiencies and antiphospholipid antibodies are seen in which of the

following?

a) Isolated decreased PT

b) Isolated decreased aPTT

c) Isolated increased PT

d) Isolated increased aPTT

5) Von Willebrand factor (vWF) is responsible for platelet adhesion, platelet-to-platelet

aggregation, and acts as a carrier for what clotting factor in the plasma?

a) Factor VIII

b) Factor VII

c) Factor IX

d) Factor X

e) Factor XI

6) Von Willebrand factor disease is the most commonly inherited bleeding disorder.

What is the genetic inheritance type?

Page 70: Quiz Hematology Oncology Part 1 of 2

Hematology & Oncology – Part 1 14Mar2009

DO NOT DISTRIBUTE - 70 -

a) X-linked

b) Y-linked

c) Autosomal dominate

d) Autosomal recessive

e) Autosomal dominant with incomplete penetrance

7.1) Which of the following is seen in von Willebrand disease?

a) Decreased factor VIII:C and decreased vWF activity

b) Decreased factor VIII:C and normal vWF activity

c) Normal factor VIII:C and decreased vWF activity

d) Normal factor VIII:C and normal vWF activity

7.2) Which of the following is seen in hemophilia?

a) Decreased factor VIII:C and decreased vWF activity

b) Decreased factor VIII:C and normal vWF activity

c) Normal factor VIII:C and decreased vWF activity

d) Normal factor VIII:C and normal vWF activity

8) Along with mildly prolonged PTT, which of the following is increased in von

Willebrand disease?

a) Factor VIII

b) vWF antigen

c) vWF activity

d) Platelet aggregation

e) Bleeding time

9) The vWF antigen and ristocetin cofactor activity (vWF:Rco) tests are diagnostic for:

a) Hemophilia A or B

b) Von Willebrand disease

c) Vitamin K deficiency

d) Bernard-Soulier disease

e) Glanzmann thrombasthenia

10) Which of the following drugs is given for mild von Willebrand disease at the time of

minor surgery, such as a dental extraction?

a) Furosemide (Lasix)

b) Desmopressin (DDAVP)

c) Alphanate (Antihemophilic Factor)

d) Benefix [coagulation factor IX (recombinant)]

e) Humate-P (antihemophilic factor [human], pasteurized)

11) Hemophilia A (factor VIII deficiency) is clinically indistinguishable from hemophilia

B (factor IX deficiency). What is their genetic inheritance pattern?

a) X-linked recessive

b) X-linked dominant

c) Y-linked

d) Autosomal recessive

e) Autosomal dominant

12) Which for of hemophilia presents clinically with infrequent spontaneous bleeding and

possible bleeding after minor trauma and surgery?

a) Mild form

b) Moderate form

Page 71: Quiz Hematology Oncology Part 1 of 2

Hematology & Oncology – Part 1 14Mar2009

DO NOT DISTRIBUTE - 71 -

c) Severe form

13) Patients with symptomatic hemophila A should initially be treated with a trial of

Desmopressin. What drug would be started for patients with hemophila B?

a) Furosemide (Lasix)

b) Desmopressin (DDAVP)

c) Alphanate (Antihemophilic Factor)

d) Benefix [coagulation factor IX (recombinant)]

e) Humate-P (antihemophilic factor [human], pasteurized)

14) A patient presents with gingival bleeding and recurrent epistaxis. Lab test show

normal aPTT but abnormal PT. Which of the following is the most likely factor that

would be deficient?

a) Factor XII

b) Factor XI

c) Factor IX

d) Factor X

e) Factor VII

15) What is the treatment of choice for factor VII deficiency?

a) Desmopressin (DDAVP)

b) Alphanate (Antihemophilic Factor)

c) Novoseven [coagulation factor VIIa (recombinant)]

d) Benefix [coagulation factor IX (recombinant)]

e) Humate-P (antihemophilic factor [human], pasteurized)

16) An Ashkanazi Jew presents with a very rare, autosomal recessive bleeding disorder.

Prior to surgery the patient is given fresh frozen plasma and a virally inactivated

concentrate of what factor, which they are deficient in?

a) Factor XII

b) Factor XI

c) Factor IX

d) Factor X

e) Factor VII

17) A child presents with marked bleeding. All bleeding tests are normal. The patient has

a history of ecchymosis and prolonged hemorrhage from cuts. Clinician initially

suspected child abuse. They believe the patient has an XIII deficiency and give

cryoprecipitate (cryo). How was this disorder passed on genetically?

a) X-linked recessive

b) X-linked dominant

c) Autosomal recessive

d) Autosomal dominant

e) It is not a genetic disorder

18) A patient presents with a bleeding disorder involving thromboembolic complications.

Lab test show a very high aPTT with a normal PT. An autosomally inherited factor

deficiency is suspected. Which deficiency is most likely?

a) Factor XII

b) Factor XI

c) Factor IX

d) Factor X

Page 72: Quiz Hematology Oncology Part 1 of 2

Hematology & Oncology – Part 1 14Mar2009

DO NOT DISTRIBUTE - 72 -

e) Factor VII

19) Which of the following is NOT a common cause of disseminated intravascular

coagulation (DIC)?

a) Malignancy

b) Infection

c) Surgery

d) Snake bite

e) Trauma

20) Thrombosis is more commonly seen than bleeding in DIC. Bleeding from what sites

is most commonly seen in DIC?

a) Perivenipuncture and mouth

b) Intraperitoneal and intrapleural

c) Skin (ecchymoses, petechiae, hematoma)

d) Vagina and in urine (hematuria)

e) Intracranial

21) In DIC, control proteins antithrombin III and protein C along with factor XII are

involved in the bleeding that is seen. In general, which of the following signifies that

hemorrhage would occur (instead of thrombosis)?

a) Plasmin activity is greater than thrombin activity

b) Thrombin activity is greater than plasmin activity

c) Plasminogen activity has stopped

d) Kallikrein activity has stopped

e) DIC is only associated with coagulation, not bleeding

22) Which of the following is NOT characteristic of DIC?

a) Thrombocytopenia

b) Decreased bleeding time

c) Increased PT

d) Increased PTT

e) Hypofibrinogenemia

23) What is the first goal in the treatment of disseminated intravascular

coagulopathy/coagulation (DIC)?

a) Administer anticoagulants

b) Administer cryoprecipitate

c) Administer platelets

d) Administer fresh frozen plasma

e) Treat the underlying disease

24.1) A patient with DIC is about to undergo surgery. Which of the following would

NOT be a pre-treatment?

a) Administer anticoagulants

b) Administer cryoprecipitate

c) Administer platelets

d) Administer fresh frozen plasma

e) Monitor platelet and fibrinogen levels

24.2) A patient with DIC is undergoing surgery and has continued bleeding. They are

started on clotting factor replacement. Which of the following should be administered as

a continuous infusion?

Page 73: Quiz Hematology Oncology Part 1 of 2

Hematology & Oncology – Part 1 14Mar2009

DO NOT DISTRIBUTE - 73 -

a) Cryoprecipitate

b) Fresh frozen plasma

c) Heparin

d) Platelets

e) Red blood cells

25) How is a diagnosis of pseudothrombocytopenia excluded?

a) CBC

b) Bleeding time

c) PT and PTT

d) Fibrin D-dimer assay

e) Peripheral blood smear

26) A patient presents with purpura, mucous membrane hemorrhage, and possible

cerebralmeningeal bleeding. History reveals a splenectomy. Lab studies show no Howell-

Jolly bodies. Thrombocytopenia is found with normal WBC and hemoglobin

concentration. Bone marrow aspiration shows a slight increase in megakaryocytes.

Platelet IgG are detected and an autoimmune disease is suspected. Which of the

following is most likely?

a) Disseminated intravascular coagulation (DIC)

b) Idiopathic thrombocytopenic purpura (ITP)

c) Thrombotic thrombocytopenic purpura (TTP)

d) Hemophilia A or B

e) Sickle cell anemia

27) What platelet count signifies the need to treat ITP if there is presence of risk factor

for bleeding, including hypertension?

a) 70-90x10^9/L

b) 50-70x10^9/L

c) 30-50x10^9/L

d) 10-30x10^9/L

e) Below 10x10^9/L

28) What is the initial treatment of choice for idiopathic thrombocytopenic purpura?

a) Heparin

b) Splenectomy

c) Immunoglobulin

d) Prednisone

e) Platelets

29) Which of the following would NOT be given to a patient with ITP and severe

bleeding?

a) Heparin

b) Platelets

c) Immunoglobulin

d) Prednisone

30) What is the treatment for steroid-refractory ITP?

a) Heparin

b) Splenectomy

c) Immunoglobulin

d) Prednisone

Page 74: Quiz Hematology Oncology Part 1 of 2

Hematology & Oncology – Part 1 14Mar2009

DO NOT DISTRIBUTE - 74 -

e) Platelets

31) Drug-induced thrombocytopenia is characterized by hemorrhagic syndrome and can

be caused by all of the following EXCEPT:

a) Warfarin

b) Heparin

c) Quinidine

d) Carbamazepine

e) TMP-SMX

32) A patient presents with acute platelet activation syndrome, with fever and chills about

5-10 minutes after an IV bolus. Lab findings show IgG antibodies to an antigen and a

platelet count 50% below normal. Diagnosis is confirmed with functional assays. What

drug is associated with this type II drug-induced thrombocytopenia?

a) Gold

b) Quinine

c) Heparin

d) Rifampin

e) Amphotericin B

33) Thrombophilia refers to:

a) A hereditary tendency for thrombi

b) An acquired tendency for thrombi

c) A tendency toward venous thrombi

d) A tendency toward arterial thrombi

e) All of the above

34) Which of the following congenital factors would NOT increase thrombosis risk?

a) Excess protein S and C

b) Prothrombin 20210A gene abnormality

c) Hyperhomocystinemia

d) Plasminogen deficiency

e) Congenital resistance of activated protein C (APC)

35) Which of the following is NOT a risk factor for thrombosis?

a) Oral contraceptive pills

b) Pregnancy

c) Diabetes mellitus

d) Obesity

e) Bradycardia

36) Along with general surgery, what type of surgery is high risk for the development of

venous thromboembolisms (VTEs)?

a) Gynecologic

b) Neurologic

c) Pediatric

d) Orthopedic

e) Urological

37) Which of the following patients has the highest risk of venous thromboembolism?

a) An obese patient with congestive heart failure in the intensive care unit

b) A pregnant non-coffee drinker who presents to the primary care clinic

c) A 50-year-old male who constantly trains for marathons and triathlons

Page 75: Quiz Hematology Oncology Part 1 of 2

Hematology & Oncology – Part 1 14Mar2009

DO NOT DISTRIBUTE - 75 -

d) A newborn with congenital defects of the soft and hard palate

e) A medical student who drinks several caffeinated beverages daily

38) Which of the following forms of venous thromboembolism (VTE) prophylaxis is

NOT recommended for high-risk patients?

a) Fondaparinux (Arixtra)

b) Unfractionated heparin (UFH)

c) Low-molecular weight heparin (LMWH)

d) Early ambulation and compression stockings

e) Warfarin, adjusted-dose

39) Factor X deficiency presents similarly to factor VII deficiency (bleeding) and may

occur in amyloidosis. Which of the following describes lab results in factor X deficiency?

a) Abnormal PT and abnormal aPTT

b) Abnormal PT and normal aPTT

c) Normal PT and abnormal aPTT

d) Normal PT and normal aPTT

40) Which of the following is used for many situations such as deficiencies in factors II,

V, VII, IX, X, XI, XIII, oral anticoagulant overdose, liver disease, and massive

transfusion?

a) Cryoprecipitate

b) Fresh frozen plasma

c) Heparin

d) Platelets

e) Red blood cells

41) Which of the following specifically contains factors V, VIII, and fibrinogen?

a) Cryoprecipitate

b) Fresh frozen plasma

c) Heparin

d) Platelets

e) Red blood cells

42) Which of the following is a clinical indicator for hypercoagulable states?

a) Family history of bleeding disorders

b) History of a surgery-related thrombosis

c) Thrombosis of a cerebral, subclavian, or mesenteric vein

d) Postpartum bleeding

e) Clotting factor deficiencies

43.1) What is the inheritance pattern for patients with resistance to activated protein C?

a) X-linked recessive

b) X-linked dominant

c) Autosomal recessive

d) Autosomal dominant

e) It is not a genetic disorder

43.2) A patient is diagnosed with resistance to activated protein C after having an aPTT-

based APC resistance ratio done. They are started on lifelong anticoagulation

prophylaxis. What factor (“Leiden”) is mutated in this disorder?

a) Factor VII

b) Factor V

Page 76: Quiz Hematology Oncology Part 1 of 2

Hematology & Oncology – Part 1 14Mar2009

DO NOT DISTRIBUTE - 76 -

c) Factor X

d) Factor XI

e) Fibrinogen

44) Protein S deficiency leads to venous complications (thrombosis) more often than

arterial. What factor does protein S depend on for expression of APC anticoagulation?

a) Von Willebrand factor

b) Vitamin K

c) Fibrinogen

d) Urokinase

e) Fibronectin

45) Protein S and C deficiencies are treated with oral anticoagulants and:

a) Cryoprecipitate

b) Warfarin

c) Heparin

d) Platelets

e) Red blood cells

46) Patients with antiphospholipid antibodies are at higher risk for thrombosis. What is

the screening test for these patients?

a) Prolonged bleeding time

b) High platelet count

c) Abnormal complete blood count (CBC)

d) Prolonged prothrombin time (PT)

e) Prolonged partial thromboplastin time (PTT)

47) Along with the platelet neutralizing procedure, what is the best confirmatory test for

the presence of antiphospholipid antibodies (Hughes syndrome), also used to test for

lupus anticoagulant?

a) Dilute Russell viper venom time (dRVVT)

b) Kaolin clotting time

c) Hexagonal phospholipid confirmatory test

d) Anti-beta2 glycoprotein I antibodies

e) Anti-phosphatidylserine antibodies,

48) Along with deep vein thrombosis (DVT), which of the following would be associated

with the presence of antiphospholipid antibodies?

a) Dysfunction platelet aggregation

b) Severe hepatosplenomegaly

c) Mucous membrane bleeding

d) Budd-Chiari syndrome

e) Fetal chorea

49) Along with hemolysis and thrombocytopenia, what hematologic complication is

associated with the presence of antiphospholipid antibodies?

a) Hypercomplementemia

b) Hypocomplementemia

c) Hypercholesterolemia

d) Hypocholesterolemia

e) Pancytopenia

50) Thrombosis in the presence of antiphospholipid antibodies is treated initial with:

Page 77: Quiz Hematology Oncology Part 1 of 2

Hematology & Oncology – Part 1 14Mar2009

DO NOT DISTRIBUTE - 77 -

a) Cryoprecipitate

b) Warfarin

c) Heparin

d) Platelets

e) Aspirin

51) Patients with a family history of thrombosis should be tested for antiphospholipid

antibodies. What other population should be tested?

a) African American older than 16

b) Patients with sickle cell anemia

c) Patients older than 40 with thrombotic events

d) Patients younger than 40 with thrombotic events

e) All hospitalized patients older than 75 years of age

52) Which of the following is a relative (not absolute) contraindication to the use of

warfarin?

a) Pregnancy

b) Thrombocytopenia

c) Eye surgery

d) CNS surgery

53) Warfarin therapy should last for 3 months in patients at low risk with INR maintained

between 2 and 3 (PT between 1.3 and 1.5). When starting anticoagulation therapy, how

soon after the initiation of heparin therapy should warfarin therapy begin?

a) Immediately

b) 1-hour

c) 24-hours

d) 72-hours

e) These drugs should not be used together

54.1) A teenager presents with suspected cerebral hemorrhage. History reveals they had

depression and attempted to overdose on their grandmother’s warfarin. What treatment

should this teenager receive?

a) Vitamin K

b) Fresh frozen plasma

c) Clotting factors (recombinant)

d) Blood transfusion

e) A & B

54.2) An elderly patient who is currently on warfarin sustains a life-threatening bleed

during surgery. What treatment should this patient receive?

a) Vitamin K

b) Fresh frozen plasma

c) Clotting factors (recombinant)

d) Blood transfusion

e) A & B

55) Which of the following drugs would NOT potentiate warfarin?

a) Phenytoin

b) Metronidazole

c) Phenylbutazone

d) Disulfiram

Page 78: Quiz Hematology Oncology Part 1 of 2

Hematology & Oncology – Part 1 14Mar2009

DO NOT DISTRIBUTE - 78 -

e) TMP-SMX

56) What is the goal of therapy in the initial treatment of deep vein thrombosis (DVT) or

pulmonary embolism (PE)?

a) Shorten PT at a level of 0.3 to 0.8 times normal within 24 hours

b) Lengthen PT at a level of 1.5 to 2.5 times normal within 24 hours

c) Shorten aPTT at a level of 0.3 to 0.8 times normal within 24 hours

d) Lengthen aPTT at a level of 1.5 to 2.5 times normal within 24 hours

e) Shorten both PT and aPTT as much as possible as soon as possible

57) How should heparin and warfarin (Coumadin) be administered when long-term

coagulation is indicated?

a) Overlapped for 24-hours then heparin should be continued

b) Overlapped for 24-hours then warfarin should be continued

c) Overlapped for 5-days then heparin should be continued

d) Overlapped for 5-days then warfarin should be continued

e) These drugs should not be given simultaneously

58) Patients undergoing abdominal surgery or with a history of thrombosis, prolonged

bed rest, congestive heart failure, or cancer should receive heparin prophylaxis of 5,000

units subcutaneously. How frequently should they receive this prophylaxis?

a) Every hour

b) Every 2-4 hours

c) Every 8-12 hours

d) Daily

e) Weekly

59) Heparin should be administered twice daily via subcutaneous injection for the

prevention of a mural thrombosis after an MI. What is the suggested dosage?

a) 1,000 units

b) 2,500 units

c) 5,000 units

d) 12,000 units

e) 23,000 units

60) Low molecular weight heparin (LMWH) is contraindicated in heparin-induced

thrombocytopenia and severe dysfunction of what organ?

a) Heart

b) Kidneys

c) Liver

d) Adrenals

e) Spleen

61) What is the primary role of anticoagulation therapy?

a) To break clots in the heart (myocardial infarction)

b) To break clots in the brain (stroke)

c) To reduce the size of femoral vein clots

d) To prevent recurrence of atrial fibrillation

e) Prophylaxis

62) Thrombolytic therapy has been shown to decrease mortality.

a) True

b) False, is has only been shown to relieve symptoms

Page 79: Quiz Hematology Oncology Part 1 of 2

Hematology & Oncology – Part 1 14Mar2009

DO NOT DISTRIBUTE - 79 -

63) Absolute contraindications for thrombolytic therapy include internal bleeding or a

cerebrovascular accident within the past two weeks, or surgery within the last:

a) Day

b) Week

c) Month

d) 6-months

e) Year

64) What is the most common cause of hemochromatosis (siderophilia)?

a) Secondary anemia and infective erythropoiesis

b) Oral intake of iron (medicinal)

c) Isoniazid or chloramphenicol

d) Copper deficiency

e) Hereditary (idiopathic)

65) Hemochromatosis can be detected with polymerase chain reaction (PCR) detection of

a mutated HFE gene on chromosome 6. What is the genetic inheritance pattern?

a) X-linked recessive

b) X-linked dominant

c) Y-linked

d) Autosomal recessive

e) Autosomal dominant

66) A 40-year-old male presents with atrial fibrillation, abdominal pain, and fatigue. The

reason for the visit is regarding his recent impotence. Blood tests reveal a transferrin

saturation greater than 50%. Since the patient is male the doctor does not bother asking

about oral contraceptives, which can increase transferrin saturation. The physician

believes the man has a genetically inherited disease. Which of the following is most

likely being stored at high volume in this patient?

a) Copper

b) Sodium

c) Iron

d) Vitamin B12

e) Folate

67) What is the diagnostic test performed for confirmation of hereditary

hemochromatosis?

a) CBC

b) Blood iron saturation

c) Bleeding and platelet count

d) Spleen biopsy

e) Liver biopsy

68) What is the treatment for hereditary hemochromatosis?

a) Phlebotomy and chelation

b) Anticoagulant therapy

c) Blood transfusion

d) Bone marrow transplantation

e) There is no current treatment

Page 80: Quiz Hematology Oncology Part 1 of 2

Hematology & Oncology – Part 1 14Mar2009

DO NOT DISTRIBUTE - 80 -

Hematology/Oncology #23 – Extra: Relieving The Pain Of Osteoarthritis

1) What joints are most commonly damaged in osteoarthritis (OA)?

a) Back, wrists, elbows

b) Shoulders, hips, ankles

c) Hips, knees, back

d) Fingers, toes

e) Neck, back

2) What is the most powerful risk factor for osteoarthritis?

a) Obesity

b) Family history

c) Repetitive joint use

d) Trauma

e) Age

3) Which of the following would be first-line therapy for OA?

a) Acetaminophen

b) Capsaicin

c) Opiates

d) Ibuprofen

e) Glucosamine

4) In a study of 50 patients over 12-weeks who had mild to moderate OA, which of the

following was seen to improve with the glucosamine group when compared to the

placebo group? (The effect of glucosamine…BrJSportsMed 2003:37(1):45-49)

a) Adverse clinical events

b) 3-meter duck walk

c) Subjective pain

d) Joint-line palpation

e) Stair climbing

5) What is considered the major event in the pathophysiology of OA?

a) Joint inflammation

b) Loss of synovial fluid

c) Increased synovial fluid viscosity

d) Joint cartilage deterioration

e) Bone micro-fractures

6) Which of the following has been shown to deplete substance P, which is implicated in

the pathogenesis of arthritis?

a) Acetaminophen

b) Capsaicin

c) Opiates

d) Ibuprofen

e) Glucosamine

7) In a trial of 130 patients, patients using chondroitin were shown to have improvements

in which of the following, when compared to placebo? (Chondroitin sulfate…JRheumatol

2001;28:173-181)

a) Adverse clinical events

b) Walking distance

c) Subjective pain

Page 81: Quiz Hematology Oncology Part 1 of 2

Hematology & Oncology – Part 1 14Mar2009

DO NOT DISTRIBUTE - 81 -

d) Impact on activities of daily living

e) All self-reported variables

8) In a study comparing capsaicin cream with placebo in both OA and rheumatoid

arthritis (RA), what was the outcome? (The effect of topical capsaicin…JRheumatol

1992;19:604-607)

a) Pain was not reduced in either

b) Pain was reduced in both

c) Pain was reduced in OA, but not RA

d) Pain was reduced in RA, but not OA

9) According to a review of seven trials representing 393 patients undergoing

acupuncture therapy for OA, which of the following is true?

a) Acupuncture showed no benefit

b) Acupuncture showed great benefit when replacing standard therapy

c) Acupuncture did not show benefit when compared with sham acupuncture

d) Acupuncture showed improvement in both pain scores and function

e) Acupuncture showed improvement in pain scores but not function

James Lamberg

Page 82: Quiz Hematology Oncology Part 1 of 2

Hematology & Oncology – Part 1 14Mar2009

DO NOT DISTRIBUTE - 82 -

AnswerKey

Heme #1

1) C

2) A

3) E

Heme #2

1) D

2) C

3.1) E

3.2) E

4.1) C

4.2) D

4.3) A

4.4) E

4.5) B

5.1) A

5.2) C

5.3) E

6) F

7.1) D

7.2) A

7.3) E

8.1) B

8.2) C

9) D

10) D

11) C

12.1) D

12.2) C

12.3) A

12.4) D

12.5) E

12.6) B

12.7) D

Heme #3

1) E

2) B

3) A

4) B

5) C

6) B

7) E

Heme #4

1) C

2) C

3) B

4) D

5) A

6) C

7) C

8.1) D

8.2) C

8.3) E

9) B

10) C

11.1) A

11.2) E

11.3) B

12.1) D

12.2) A

12.3) C

12.4) B

12.5) E

13) B

Heme #5

1) D

2) A

Heme #6

1.1) C

1.2) D

1.3) A

1.4) E

1.5) C

2) B

3.1) A

3.2) D

4.1) C

4.2) A

4.3) E

5) D

6) D

7.1) E

7.2) C

7.3) C

7.4) E

7.5) E

Heme #7

1) C

2) E

3) C

4.1) B

4.2) C

4.3) A

4.4) D

5) E

6) B

7.1) B

7.2) C

8.1) E

8.2) E

8.3) A

8.4) A

Heme #8

1) C

2) C

3.1) A

3.2) D

4) B

5) C

6.1) C

6.2) E

6.3) A

7) A

8.1) B

8.2) E

9.1) E

9.2) C

10.1) E

10.2) D

11) D

12) A

13) A

14) D

15) E

16.1) E

16.2) A

17.1) A

17.2) C

17.3) C

17.4) D

18) E

19.1) A

19.2) C

19.3) D

19.4) B

Heme #9

1.1) C

1.2) A

1.3) E

1.4) E

1.5) B

1.6) C

2) E

3.1) C

3.2) A

4) C

5) E

6) C

7) A

8) D

9.1) D

9.2) E

10) B

11.1) D

11.2) C

11.3) A

11.4) A

11.5) D

11.6) B

Heme #10

1.1) C

1.2) E

2) E

3) D

4.1) A

4.2) C

5) C

6) B

7) A

Heme #11

1) C

2) E

3) D

4) B

5) B

6) A

7) D

Heme #12

1) C

2) D

3) A

4.1) B

4.2) C

4.3) D

5) A

6) D

7) C

8) B

9) F

10) A

11.1) B

11.2) D

12) E

13) C

Heme #13

1) D

2) B

3) A

4) B

5) C

6) D

Heme #14

1) D

2.1) A

2.2) D

3) D

4) B

5) E

6) A

7) C

8) D

9) C

10) D

11) D

12) B

13) E

14) A

Page 83: Quiz Hematology Oncology Part 1 of 2

Hematology & Oncology – Part 1 14Mar2009

DO NOT DISTRIBUTE - 83 -

15) B

Heme #15

1) B

2) A

3) C

4) B

5) D

6) E

7) D

8) D

9) E

10) D

11) E

12) E

Heme #16

1) C

2) A

3) B

4) C

5) B

Heme #17

1) D

2) B

3) E

4) C

5) A

6) B

7) D

8) E

9) D

10) A

11) C

12) D

13) B

14) C

15) E

Heme #18

1) C

2) E

3) D

4) A

5) B

6) A

7) A

8) B

9) D

10) D

11) E

12) A

13) B

Heme #19

1) D

2) E

3) A

4) B

5) C

6) A

7) C

8) D

9) E

10) C

11.1) B

11.2) C

12) A

13) B

14) D

15.1) C

15.2) C

16) D

17.1) E

17.2) C

17.3) B

17.4) A

18) B

19) E

20) C

21) A

22) B

23) D

24.1) B

24.2) E

24.3) D

24.4) C

25) C

26) C

27) D

28) E

29.1) B

29.2) A

30) C

31) C

32) D

33) B

34) D

35.1) C

35.2) D

36) C

37) A

38) B

39) E

40) D

Heme #20

1) D

2) B

3) A

4) E

5) C

6) B

7) D

8) A

9) C

10) B

11) D

12) A

13) B

14) E

15) B

16) A

17) C

18) D

19) A

20) E

21) B

22) C

23) A

24) B

25) A

26) A

27) D

28) D

29) B

30) D

31) C

Heme #21

1) C

2) C

3) C

4) E

5) D

6) A

7) B

8) C

9) D

10) D

11) E

12.1) E

12.2) B

12.3) A

13) E

14.1) C

14.2) A

15) D

16) C

17) B

18) C

19) C

20) D

21.1) C

21.2) D

22) F

23) D

24) B

25) A

26) E

27) D

28) C

29) B

30) E

31) D

32) A

33) E

34) C

35) B

36) E

37) B

38) E

39) A

40) C

41) A

42) D

43) E

44) C

45) E

46) A

47) D

48) A

49) D

50) D

51) C

52) D

53) E

54) E

55.1) A

55.2) D

56) A

57) C

58) B

59) E

60) D

61) B

62) E

63) A

64) E

65) D

66) B

67) C

68) C

69) E

70) E

71) D

72) A

73) D

74) E

75) B

76) C

77) A

78) B

79) D

80) E

81) E

82) C

83) B

84) A

Page 84: Quiz Hematology Oncology Part 1 of 2

Hematology & Oncology – Part 1 14Mar2009

DO NOT DISTRIBUTE - 84 -

85) B

86) C

87) D

88) E

89.1) A

89.2) B

90) D

91) E

92) B

93) D

Heme #22

1) E

2) D

3) C

4) D

5) A

6) E

7.1) A

7.2) B

8) E

9) B

10) B

11) A

12) B

13) D

14) E

15) C

16) B

17) C

18) A

19) D

20) A

21) A

22) B

23) E

24.1) A

24.2) C

25) E

26) B

27) C

28) D

29) A

30) B

31) A

32) C

33) E

34) A

35) E

36) D

37) A

38) B

39) C

40) B

41) A

42) C

43.1) D

43.2) B

44) B

45) C

46) E

47) A

48) D

49) B

50) C

51) D

52) B

53) C

54.1) E

54.2) B

55) A

56) D

57) D

58) C

59) D

60) B

61) E

62) A

63) C

64) E

65) D

66) C

67) E

68) A

Heme #23

1) C

2) E

3) A

4) C

5) D

6) B

7) E

8) C

9) E